NCLEX RN > EXAM > NCLEX-RN U-WORLD REVIEW 680 Questions with Verified Answers,100% CORRECT (All)

NCLEX-RN U-WORLD REVIEW 680 Questions with Verified Answers,100% CORRECT

Document Content and Description Below

NCLEX-RN U-WORLD REVIEW 680 Questions with Verified Answers Allen's test - CORRECT ANSWER This diagnostic test is performed prior to initiating radial cannulation to ensure adequate circulation pro... ximal and distal to the site Agoraphobia - CORRECT ANSWER Defined as the fear or anxiety about being in (or anticipating) certain situations or physical spaces; Ex- being on a train for an extended period of time Delirium - CORRECT ANSWER Defined as an acute onset cognitive dysfunction where consciousness is impaired, memory may be impaired and is reversible; Seen in ICU's Dementia - CORRECT ANSWER Defined as a chronic, gradual condition which causes irreversible and progressive cognitive decline; LOC intact, some memory loss is present and is irreversible Depression - CORRECT ANSWER Defined as a disease with gradual progression, may be episodic & is reversible - Providing a calming environment with minimal stimulation - Private room AWAY from nurses station is key - Use quiet/monotone voice - Use eye contact/gestures carefully - Limit visual clutter - Maintain minimal lighting - Provide child with single object to focus on - CORRECT ANSWER When caring for a child with autism spectrum disorder, what are nursing actions/interactions that should be implemented? Autism Spectrum Disorder (ASD) - CORRECT ANSWER Patients with this cognitive disorder often exhibit sensory processing problems to sounds, movement, touch, taste and smells Oligohydramnios - CORRECT ANSWER A condition characterized by low amniotic fluid volume Pulmonary hypoplasia & umbilical cord compression - CORRECT ANSWER What are two major complications of oligohydramnios at time of birth? Pulmonary hypoplasia - CORRECT ANSWER A complication of oligohydramnios defined as lack of normal alveolar dissension by aspirated amniotic fluid Umbilical cord compression - CORRECT ANSWER A complication of oligohydramnios which can present as late/variable decelerations; Requires continuous fetal monitoring at birth Polyhydramnios - CORRECT ANSWER Defined as a complication of excessive amniotic fluid volume, putting the mother at risk for post partum hemorrhage Pull the pinna upward & back to straighten the external ear canal - CORRECT ANSWER When administering an otic medication to an adult or child age 3 and older, how should the nurse manipulate the pinna? Instilling cold drops into the ear can cause vestibular reaction, resulting in dizziness/vomiting - CORRECT ANSWER Why should otic medication be warmed to room temperature before administration? ACE inhibitors - CORRECT ANSWER What class of blood pressure medications is category X in pregnancy and are extremely teratogenic? Placental abruption (Placento abrupto) - CORRECT ANSWER Defined as a complication of preeclampsia that occurs when the placenta tears away from the wall of the uterus due to stress, causing significant vaginal bleeding HELLP syndrome: H - Hemolysis EL - Elevated liver enzymes LP - Low platelets - CORRECT ANSWER A complication of pregnancy defined as severe eclampsia; What does this acronym stand for? At least 0.5 mL/kg/hr or approximately 30 mL/hr - CORRECT ANSWER What is normal urine output for an adult? DKA - Diabetic Ketoacidosis - CORRECT ANSWER An endocrine disorder characterized by breakdown of fat for fuel and the resulting byproducts are acidic ketones; Polyuria & kussmaul respirations are present to help blow off extra co2 Palpation - CORRECT ANSWER This disorder of pediatrics presents as an abdominal pass. What nursing intervention is PROHIBITED with this diagnosis? Sickle cell anemia - CORRECT ANSWER This disorder can present as a vasoocclusive crisis leading to ischemia and severe pain; Bilirubin released from the destroyed RBC's leads to jaundice and brownish hue to urine Codeine - CORRECT ANSWER This medication is an opioid drug prescribed to treat mild/moderate pain & is also an antitussive to suppress the cough reflex Allopurinol - CORRECT ANSWER This medication is prescribed to prevent gout attacks characterized by pain/inflammation in the joints due to uric acid deposits; Will need to be taken for the rest of life Allopurinol should be taken with a FULL glass of water & overall fluid intake should be increased when taking this medication - CORRECT ANSWER What is the MOST important teaching topic when educating a patient about administration of allopurinol? Kawasaki Disease (KD) - CORRECT ANSWER This childhood condition is defined as inflammation of arterioles, also known as vasculitis Coronary aneurysms - CORRECT ANSWER For pediatric patients diagnosed with Kawasaki Disease (KD), what are they most at risk for developing? Acute phase - CORRECT ANSWER This phase of Kawasaki Disease is characterized by acute, sudden onset of high fever that does not respond to antibiotics or antipyretics; Strawberry tongue develops Subacute phase - CORRECT ANSWER This phase of Kawasaki Disease is characterized by peeling of skin on hands and feet Convalescent phase - CORRECT ANSWER This phase of Kawasaki Disease begins when the symptoms begin to slowly disappear; Temperature returns to normal IV immunoglobulin (IVIG) and high dose aspirin - CORRECT ANSWER What is the initial treatment of Kawasaki Disease? 46-70 seconds (1.5-2x control) - CORRECT ANSWER What is the therapeutic aPTT value for a client receiving a continuous heparin infusion? Warfarin - Vitamin K Heparin - Protamine sulfate - CORRECT ANSWER What are the reversal agents for warfarin & heparin? Water intoxication resulting in hyponatremia - CORRECT ANSWER This occurs when parents of newborns dilute formula to "stretch" the feeding and save money Irritability, lethargy, hypothermia and seizure activity - CORRECT ANSWER What are some s/s of hyponatremia? "Airing MTV:" - Measles - Tuberculosis - Varicella - CORRECT ANSWER What diseases require airborne precaution in the hospital? A private room with negative airflow pressure - CORRECT ANSWER For patients under airborne isolation precaution, what kind of room placement is required? Intussusception - CORRECT ANSWER Defined as a process in which one part of the intestine prolapses and then telescopes into another part; Very common during infancy - Legs drawn up toward abdomen - Continuous crying due to pain - Grossly bloody "Currant jelly" stools (Mixture of blood and mucus) - CORRECT ANSWER What would the clinical manifestations be for an infant suspected of having intussusception? An air (Pneumatic) enema - CORRECT ANSWER What is the treatment for intussusception? Nephrotic Syndrome - CORRECT ANSWER This disease is defined as a collection of symptoms resulting from various causes of glomerular injury - Massive proteinuria*** - Hypoalbuminemia in serum (Massive loss in urine) - Edema (Periorbital and peripheral) - Hyperlipidemia - CORRECT ANSWER What are the 2 classic manifestations of nephrotic syndrome? < 200 mg/dL - CORRECT ANSWER What is the normal level of total cholesterol? 3.5-5.0 (Same as K) - CORRECT ANSWER What is normal serum albumin? - Personal/family history of colorectal cancer/polyps - Personal history of IBS, Crohn's or UC - Lifestyle factors such as obesity, smoking, alcohol consumption and diet high in red meat - CORRECT ANSWER What are examples of risk factors of developing colorectal cancer? External fixator - CORRECT ANSWER For patients with musculoskeletal issues, this device is a metallic device composed of metal pins, which are screwed into the bone to stabilize it Wound dehiscence & evisceration - CORRECT ANSWER For patients that underwent open gastric surgery, what are examples of complications that they are at risk for post-op? Obese (Bariatric) patients - CORRECT ANSWER Which clients that underwent open gastric surgery are at GREATEST risk for dehiscence and evisceration? - Furosemide (Lasix) - Tursemide (Demadex) - Bumentanide (Bumex) - CORRECT ANSWER What are 3 examples of loop diuretics that may be prescribed for patients with CHF? ARB inhibitors - CORRECT ANSWER This class of blood pressure medications are prescribed for patients that cannot tolerate ACE inhibitors. They block the angiotensis-aldosterone system Transfusion reaction - CORRECT ANSWER SOB, fever, low back pain, flushing & itching upon initiation of transfusion of a blood product indicate what? - Stop transfusion ASAP and disconnect tubing at catheter hub - Maintain IV access with NS to prevent vascular collapse - Notify HCP and blood bank - Monitor vital signs - CORRECT ANSWER After recognizing that a patient is experiencing a transfusion/hemolytic reaction to a blood product, what is the nurses next actions? 2.0-3.0; Up to 3.5 for prosthetic heart valves/valve disease - CORRECT ANSWER For patients prescribed warfarin, what is the therapeutic value for their INR? Meningitis - CORRECT ANSWER Defined as inflammation of the meninges of the brain and spinal cord caused by infection - Fever - Restlessness - High pitched cry - Bulging/tense fontanels (Due to hydrocephalus and increased ICP - Increased head circumference - CORRECT ANSWER What are the clinical manifestations of meningitis in an infant? Pulse pressure - CORRECT ANSWER Defined as the difference between systolic and diastolic blood pressures Systolic HTN, widened pulse pressure, bradycardia, altered respirations (Hypo or hyper) - CORRECT ANSWER What does Cushing's triad consist of r/t increased ICP? The patients farthest away from the exposure site because they are most salvageable - CORRECT ANSWER For patients suspected of being exposed to radiation, when triaging, what victims should the RN assess first? Botulism - Caused by clostridium botulinum - CORRECT ANSWER Clinical manifestations of this biological weapon include: - Symmetrical descending flaccid paralysis - Cranial nerve palsies (Ptosis, diplopia, dysphagia, dysphonia) - Shakiness - Palpitations - Nervousness - Diaphoresis - Anxiety - Hunger - Pallor - Change in LOC - CORRECT ANSWER What are the classic s/s of hypoglycemia? 15-20 grams of a quick acting carbohydrate: - 4-6 ox regular soda - 8-10 lifesaver candies - 1 tbs honey or syrup - 4 tsp jelly - 4-6 oz orange juice - 8 oz low fat milk - Commercial dextrose products - CORRECT ANSWER What is the appropriate treatment for hypoglycemia for an alert & oriented patient? Subq or IM injection of 1 mg glucagon; Alternate treatment of 25-50 mL of 50% glucose IV - CORRECT ANSWER What is the appropriate treatment for hypoglycemia for an unconscious patient? Uterine rupture - CORRECT ANSWER For patients with a hx of c-section that attempt a vaginal birth (VBAC), what are they at highest risk for? First sign: Abnormal fetal heart rate patterns Other manifestations: Constant abdominal pain, loss of fetal station, sudden cessation of uterine contractions - CORRECT ANSWER What is the first sign of uterine rupture? What are other clinical manifestations? Magnetic resonance cholangiopancreatography (MRCP) - CORRECT ANSWER This diagnostic test is a non-invasive test used to visualize the biliary and hepatic ducts via MRI - Presence of certain metal implants (Pacemaker, aneurysm clip, cochlear implant) - PREGNANCY - Allergy to gadolinium (Contrast media used) - Most orthopedic implants are safe - CORRECT ANSWER For patients scheduled to undergo a magnetic resonance cholangiopancreatography (MRCP), what are some contraindications to this procedure? Botulism - CORRECT ANSWER This neurotoxin is found in soil and can grow in any food contaminated with the spores; Main source is improperly canned or stored foods -Raw (Wild) honey - CORRECT ANSWER Ingestion of what food can potentially cause infection with botulism in infants? - Prodromal phase - Aural phase - Ictal phase - Postictal phase - CORRECT ANSWER What are the 4 phases of seizures? Prodromal phase - CORRECT ANSWER This phase of seizures is the period with warning signs that precede the seizure Aural phase - CORRECT ANSWER This phase of seizure is the period before the seizure occurs when the client experiences visual or other sensory changes Ictal phase - CORRECT ANSWER This phase of seizures is the period of active seizure activity Postictal phase - CORRECT ANSWER This phase of seizures is where the client may experience confusion from recovering - AVOID HIP ADDUCTION AND FLEXION - Place abductor wedge pillow between the legs/knees when turning client from side to side - Educate the pt not to cross legs at ankles OR knees to avoid adduction - HOB should be maintained at < 45 degrees - Educate pt that they should not sleep or be position on AFFECTED side unless directed by HCP - CORRECT ANSWER When caring for a patient post total hip arthroplasty, what are proper nursing interventions? Malignant hyperthermia (MH) - CORRECT ANSWER Defined as a rare but life threatening inherited muscle abnormality that is triggered by specific inhaled anesthetic agents and the depolarizing muscle relaxant succinlcholine to induce general anesthesia Calcium - CORRECT ANSWER For patients experiencing malignant hyperthermia due to depolarizing muscular blocker, Succinylcholine, excessive release of which element from the muscles causes sustained muscle rigidity and contraction? - Jaw/upper body rigidity (early sign) - Increased oxygen demand and metabolism - Dangerously high temperature (later sign) - CORRECT ANSWER What are the clinical manifestations of malignant hyperthermia? Creatinine clearance (Cr) - CORRECT ANSWER Defined as a measure of glomerular function and is a sensitive indicator of renal disease progression - A 24 HOUR urine collection is necessary for this test - First urine specimen is discarded and time is noted - All other voided urine in next 24 hours is collected and kept cool (On ice or refrigerated) - Blood is drawn to measure serum Cr in addition to urine Cr - CORRECT ANSWER How is creatinine clearance measured? Calcium scoring CT exam; No preparation is required - CORRECT ANSWER This diagnostic test is a noninvasive procedure that is performed to visualize calcification of the coronary arteries and heart valves. What type of client preparation is required? 1.010-1.025 - CORRECT ANSWER What is normal urine specific gravity? Suction control chamber - CORRECT ANSWER When caring for a patient with a chest tube, where can the nurse expect continuous, gentle bubbling in the collection device? Water seal chamber - CORRECT ANSWER When caring for a patient with a chest tube, where can the nurse expect tidaling to be observed? The lung has re-expanded - CORRECT ANSWER When caring for a patient with a chest tube, the nurse notices that tidaling in the water seal chamber has ceased. What has most likely occurred? - Briefly soak area in cool water to stop the burning process - Remove any clothing or jewelry around the era - Cover with a clean, dry cloth to prevent contamination and further trauma - CORRECT ANSWER What home care interventions should the RN teach a patient r/t minor burns? - Do not apply ice directly to burn as this will cause further tissue damage - Do not apply ointments, creams or - CORRECT ANSWER When educating a patient on home care for minor burns, what should the nurse instruct the patient NOT to do? Before: - Nurse must witness informed consent - D/c all anticoagulants and anti platelet agents for at least one week - Client should be typed/crossmatched for blood in case of a hemorrhage situation After: - Nurse should monitor vitals for at least every 15 min for s/s of hypovolemia due to bleeding - Assess puncture site for bleeding - CORRECT ANSWER For a patient scheduled to undergo a percutaneous left kidney biopsy, what are proper nursing interventions for before/after procedure? - Antipsychotics - Anticholinergics - Antihistamines - Antihypertensives - Benzos - Diuretics - Opioids - Sliding scale insulins - CORRECT ANSWER Related to the Beers criteria, what are classes of medications that should be avoided or administered in caution in the elderly? Aortic dissection - CORRECT ANSWER This complication of HTN occurs when the arterial wall intimal layer tears and allows blood between the inner and middle layers; EMERGENCY* - Abrupt onset and "worst ever", "tearing," or "ripping," pain with HTN being a contributing factor - CORRECT ANSWER What clinical manifestations would a patient report suspected of having an aortic dissection? - STOP INFUSION & CALL FOR HELP* - Ensure patent airway & administer o2 via high flow non-rebreather - Give IM epinephrine (Bronchodilation and vasoconstriction) - Administer adjunctive therapies (Corticosteroids, bronchodilators, antihistamines) - Monitor and reassess vitals for changes - Notify HCP - CORRECT ANSWER After administration of an antibiotic, the patient develops SOB and parities. What are proper nursing interventions when caring for a patient experiencing anaphylaxis? Gloves should be worn when applying the medication to prevent the spread of - CORRECT ANSWER When educating a patient with active genital herpes infection about application of acyclovir topical cream, how should the patient apply the cream? At 1 year of age - CORRECT ANSWER When assessing the babinski reflex in newborns, when will this reflex disappear? Epstein's pearls - CORRECT ANSWER Defined as white, pearl like epithelial cysts on gum margins and the palate in newborns Within a few weeks of birth - CORRECT ANSWER When do Epstein's pearls disappear in newborns? 2 arteries & 1 vein - CORRECT ANSWER When assessing the umbilical cord of a newborn, how many arteries/veins should be present? Suctioning - This will deplete the client's oxygen level and cause inaccurate results - CORRECT ANSWER When performing an ABG on a sedated, ventilated patient, what nursing intervention should be avoided prior which could skew the results? Greater than or equal to 65 mm Hg - CORRECT ANSWER What is normal MAP (Mean arterial pressure)? Extravasation - CORRECT ANSWER A complication of IV therapy defined as infiltration of a drug into the tissue surrounding the vein Discontinue the infusion ASAP - Disconnect the IV tubing - Aspirate the drug with a syringe - Elevate extremity above heart - Notify HCP and prepare for administration of antidote, phentolamine - CORRECT ANSWER A patient receiving norepinephrine via peripheral IV catheter is suspected of extravasating. What are the nurse's first actions? Phentolamine (Regitine) - A vasodilator that counteracts effects of some adrenergic agonists (Norepi, dopamine) - CORRECT ANSWER What is the anecdote for norepinephrine? What class of medications is this? E. coli - CORRECT ANSWER What bacteria are UTI's most commonly caused by? Cystitis - lower urinary tract infection that involves inflammation of the bladder mucosa - CORRECT ANSWER For patients diagnosed with a UTI, what is a complication of a UTI that may be present if left untreated? - Burning with urination (Dysuria) - Urinary frequency - Urinary urgency - Hematuria - Suprapubic discomfort - CORRECT ANSWER What are the clinical manifestations of cystitis (Lower UTI)? - Development of N&V - Present fever and chills - Flank pain - Costovertebral angle tenderness - CORRECT ANSWER For a patient diagnosed with a UTI, what assessment findings would be indicative that this patient developed pyelonephritis? Cirrhosis - CORRECT ANSWER Defined as a progressive, degenerative disease caused by destruction and subsequent disordered regeneration of the liver parenchyma; Associated diseases include ascites, varices and encephalopathy Hypotension, bradycardia and pink/dry skin due to vasodilation - CORRECT ANSWER What are the classic signs of neurogenic shock? - Amenorrhea - N&V - Urinary frequency - Breast tenderness - Quickening - Excessive fatigue - CORRECT ANSWER What are examples of presumptive (Subjective) signs of pregnancy? - Uterine/cervical changes (Goodell's, chadwick's, hegar, uterine enlargement) - Braxton hicks contractions - Ballottement - Fetal outline palpation - Uterine & funic souffle - Skin pigmentation changes (Linea nigra, cholasma, areola darkening) - Striae gravidarum (stretch marks) - Positive pregnancy test - CORRECT ANSWER What are examples of probable (Objective) signs of pregnancy? - Fetal heartbeat with doppler device - Fetal movement palpated by HCP - Visualization of fetus by ultrasound - CORRECT ANSWER What are examples of positive (diagnostic) signs of pregnancy? Scabies - CORRECT ANSWER This skin infection is caused by Sarcoptes scabies mite Person-to-person contact - CORRECT ANSWER How is scabies infection spread? - Treat all persons who have had contact with the infested child - Apply permethrin cream to all areas of body below the head - Place clothes, linens and stuffed animals in a plastic bar or wash them in hot water with hottest drying cycle - CORRECT ANSWER What is proper treatment regimen when a child in a household is diagnosed with scabies? NSAIDs and BBs NSAIDs - such as Motrin can cause hypersensitivity in asthmatics which can cause bronchospasm BB - Non-cardioselective beta blockers block beta cells in heart and lungs, leading to pulmonary compromise in asthmatics - CORRECT ANSWER What two classes of commonly used drugs can cause problems for clients with asthma? Hypospadias - CORRECT ANSWER Defined as a cogenital defect in which the urethral opening os on the underside of the penis Presence of urine output, indicating urethral patency - CORRECT ANSWER For a male infant post op hypospadias repair, what is most important for the nurse to monitor for? - Hypovolemia - Hypoxia - Hydrogen ions (Acidosis) - Hypokalemia or hyperkalemia - Hypothermia - CORRECT ANSWER What are the 5 H's that are reversible causes of asystole/PEA? - Tension pneumothorax - Tamponade, cardiac - Toxins (Benzo, narcotics) - Thrombosis (Pulmonary/coronary) - Trauma - CORRECT ANSWER What are the 5 T's that are reversible causes of asystole/PEA? Pulseless electrical activity (PEA) - CORRECT ANSWER On the EKG, this rhythm occurs when the cardiac monitor shows organized electrical activity but there is not adequate mechanical activity of the heart muscle Ventricular septal defect - CORRECT ANSWER Defined as an acyanotic congenital heart defect causing blood to shunt from the left side of the heart to the right, resulting in pulmonary hypertension Aspiration and inadequate nutrition to to eating/feeding difficulties - CORRECT ANSWER What is a child diagnosed with a cleft palate at risk for related to feeding? - Hold infant upright - Tilt bottle so that nipple is always filled with formula - Keep nipple down and away from cleft - Use special bottles and nipples - Burp infants often to avoid stomach dissension and regurgitation - Feed slowly over 20-30 minutes to prevent risk of aspiration - Feed q3-4hr - CORRECT ANSWER What is proper feeding technique for an infant with a cleft palate? - Re-experiencing traumatic event (Dreams, flashbacks, pounding HR, diaphoresis) - Avoiding reminders of trauma (Inability to set goals, amnesia) - Increased anxiety and emotional arousal (Insomnia, rage, fear, difficulty concentration, exaggerated startle response) - CORRECT ANSWER What are examples of reports of patients with PTSD? Fidelity - CORRECT ANSWER Ethical principle defined as exhibiting loyalty and fulfilling commitments made to oneself and others Accountability - CORRECT ANSWER Ethical principle defined as taking responsibility for ones actions Nonmaleficence - CORRECT ANSWER Ethical principle defined as doing no harm and protecting clients from danger when they are unable to do so themselves due to mental/physical condition Autonomy - CORRECT ANSWER Ethical principle defined as the patients right to make decisions for oneself; Ex: Informed consent Beneficence - CORRECT ANSWER Ethical principle defined as "Doing good!" Justice - CORRECT ANSWER Ethical principle defined as treating all clients fairly and without bias Veracity - CORRECT ANSWER Ethical principle defined as telling the truth as a fundamental part of building a trusting relationship 10-20 mcg/mL - CORRECT ANSWER What is the therapeutic index for theophylline? > 20 mcg/mL - CORRECT ANSWER At what level does theophylline toxicity occur? Isoniazid (INH) - CORRECT ANSWER This medication is the firs tline anti tubercular drug prescribed to treat TB Hepatotoxicity and peripheral neuropathy - Educate pt to report yellowing of skin/sclera or numbness and tingling of extremities - CORRECT ANSWER What two serious adverse effects are possible with Isoniazid (INR) therapy? What should the RN instruct the patient to report? Alcohol - INH causes hepatotoxicity - CORRECT ANSWER For patients taking Isoniazid (INH) to treat TB, what beverage should the RN educate the patient to AVOID? Thiamine (Vitamin B1) - CORRECT ANSWER For clients with history of alcoholism, what vitamin should be administered to prevent Wernickle encephalopahy? Alcoholism - Lack of thiamine (vitamin b12) - CORRECT ANSWER Wernickle encephalopathy is caused by what chronic condition and lack of what vitamin? 39-50% - CORRECT ANSWER What is the average normal hematocrit value? 13.2-17.3 g/dL - CORRECT ANSWER What is the average normal hemoglobin value? - Avoid excessive caffeine and alcohol - Never abruptly discontinue anti seizure medications - Perform adequate oral hygiene (Gingival hyperplasia) - Use alternative birth control methods other than PO meds - Wear a medic alert bracelet identifying seizure hx - CORRECT ANSWER For patients diagnosed with epilepsy being treated with Phenytoin, what are nursing education points the RN should teach the patient? - Brush after meals - Floss at least BID - Rinse mouth with water after meals if brushing isn't possible - Chew sugar free gum instead of sugary gum - Avoid simple sugars, sticky goods, sugary beverages and other sodas and fruit drinks - CORRECT ANSWER What nursing education points should be given when teaching on how to prevent dental caries? Hematuria, hypotension and diuresis - CORRECT ANSWER When catheterizing a patient with acute urinary retention, what can rapid decompression of the bladder cause? Carbon monoxide (CO) - CORRECT ANSWER Defined as a toxic inhalant that enters the blood and is more readily available to hemoglobin than oxygen Falsely high - Pulse oximeters cannot differentiate between CO and o2 - CORRECT ANSWER When hemoglobin is saturated with CO, how will the pulse oximeter read? - Headache - Dizziness - Fatigue - Nausea - Dyspnea - Normal pulse ox reading - CORRECT ANSWER For patients suspected of CO poisoning, what kinds of nonspecific symptoms will be present? - Non smokers: < 5% - Smokers: < 10% - CORRECT ANSWER What is the normal serum carboxyhemoglobin level for non smokers? Smokers? It is an expected finding - Normal - CORRECT ANSWER For patients that underwent a previous mastectomy, what does a small, firm and non-tender axillary lymph node indicate? Malignancy - Cancer - CORRECT ANSWER What does a tender, hard, fixed or enlarged axillary lymph node possibly indicate? - C: Change in bowel or bladder habits (Black stool, painless hematuria) - A: A sore that does not heal - U: Unusual bleeding/discharge (From vagina, cervix or endometrium) - T: Thickening or lump in breast or elsewhere - I: Indigestion or difficulty swallowing - O: Obvious change in wart/mole - N: Nagging cough or hoarseness (Lung cancer, larynx) - CORRECT ANSWER What does the mnemonic CAUTION mean when educating patients on s/s of possible malignancy? Rhabdomyolysis - CORRECT ANSWER This condition occurs when muscle fibers are realized into the blood usually after an intense muscle injury from exercise, heat stroke or physical trauma - Oliguria - Dark amber urine (Myoglobinuria) - Muscle pain - CORRECT ANSWER For patients suspected of having rhabdomyolysis, what clinical manifestations would be present? - Fluid resuscitation (Prevent kidney damage from elevated myoglobin) - Urine sample (Myoglobinuria) - ECG (Hyperkalemia) - IV morphine - CORRECT ANSWER For patients diagnosed with rhabdomyolysis, what are vital nursing interventions to implement? - PT/INR and aPTT due to increased bleeding risk - CORRECT ANSWER Prior to liver biopsy, what laboratory values does the nurse need to check and why? CN IX: Glossopharyngeal - CORRECT ANSWER This CN is involved in the gag reflex, ability to swallow, phonation and taste Ca: - Fish - Sardines, salmon, trout - Tofu - Green vegetables - Spinach, kale, broccoli - Almonds Vitamin D: - Egg yolks - Oily fish: Salmon, sardines, tuna - CORRECT ANSWER For patients who are lactose intolerant, what foods will these patients receive adequate calcium and vitamin D from? Transition phase - CORRECT ANSWER Presentation of mucous, and pink/dark brown blood is present during this phase of labor; Aka bloody show Once complete dilation (10cm) is achieved - CORRECT ANSWER When does the second stage of labor begin? - Bathe patients with MRSA with pre-moistened cloths or warm water containing chlorhexidine solution - CORRECT ANSWER For clients in the ICU diagnosed MRSA on the skin, what is the recommended bathing procedure for the UAP? - Apnetic and cyanotic during feeding - Frothy saliva, coughing, choking & drooling - At greatest risk for aspiration! - CORRECT ANSWER For newborns suspected of having esophageal atresia with tracheoesophageal fistula, how would this newborn most likely present? What are they at greatest risk for? Priapism - CORRECT ANSWER Defined as a prolonged, painful erection (>2 hours) caused by trapping blood in the penile vasculature that can lead to erectile tissue hypoxia and necrosis These patients require "Permissive hypertension" during the first 24-49 hours to allow for adequate perfusion through the damaged cerebral tissues - CORRECT ANSWER For a patient with an acute stroke presentation, what type of blood pressure management is required and for how long? - Ecchymosis and tenderness over the thigh/hip - Groin/hip pain with weight baring - Muscle spasm in the injured area - Shortening of the affected extremity* - Abduction or adduction of the affected extremity - Affected extremity usually externally rotated* - CORRECT ANSWER What are the most common clinical manifestations of hip fractures? Age 21 - CORRECT ANSWER When is screening for cervical cancer in women begin? Antihistamines & corticosteroids - CORRECT ANSWER For patients undergoing allergy skin testing, what class of medications needs to be withheld prior? Testicular torsion - CORRECT ANSWER Emergency medical condition in which the blood flow to the testis (scrotum) has stopped; The testicle rotates and twists the spermatic cord, initially causing obstruction that leads to severe pain & will lead to testicular ischemia and necrosis Osteoarthritis (OA) - CORRECT ANSWER This disorder is also called degenerative joint disease, seen in knees, hips and fingers - Pain - Exacerbated by weight baring - Crepitus: Grating noise/sensation with movement that can be palpated - Morning stiffness - Decreased ROM/mobility - Atrophy of the muscles that support the joint (Quads, hamstrings) - CORRECT ANSWER What are the clinical manifestations of OA in the knee joint? Ventricular rate decreased from 158/min to 88/min - CORRECT ANSWER For patients with a diagnosis of atrial fibrillation prescribed Cardizem, what client outcome best indicates that the medication has taken its intended effect? - Withdrawn behavior - Monotone speech that is difficult to understand - Speaks with a loud voice - CORRECT ANSWER What clinical manifestations will be present in a toddler suspected of having hearing deficit? Spina bifida - CORRECT ANSWER This congenital defect is characterized in which the spinal cord contents can protrude through the vertebrae that did not close A tuft of hair or a hemangioma at site - CORRECT ANSWER Upon examining a newborn, the nurse suspects spina bifida a when she assess which finding? Folic acid - CORRECT ANSWER Deficit of which supplement can lead to neural tube defects in newborns? Mongolian spots - Congenital dermal melanocytosis - CORRECT ANSWER Defined as bluish discolored areas on the lower back and or buttock, most commonly seen in african american, asian, hispanic and native american infants - Older adults - Patients with suppressed immunity - Long history of antibiotic use - Invasive tubes/lines - Patients in ICU - Patients on ventilators - CORRECT ANSWER What are examples of clients that are at highest risk for MRSA? Hyperemesis gravidarum - CORRECT ANSWER During pregnancy, this disorder causes pregnancy clients severe nausea and vomiting leading to fluid and electrolyte imbalances, nutritional deficiencies, ketonuria and weight loss - Positive urine ketones - BP/HR changes for hypovolemia (Tachycardia, hypotension) - Poor skin turgor - Delayed capillary refill - Urine specific gravity > 1.025 due to concentrated urine - BUN > 20 - CORRECT ANSWER What are the clinical manifestations of hyperemesis gravidarum? Placenta previa - CORRECT ANSWER During pregnancy, defined as an abnormal implantation of the placenta resulting in partial or complete covering of the cervical os (opening) Placenta previa - CORRECT ANSWER A woman at 20 weeks of gestation reports mild contractions & painless vaginal bleeding. What should the nurse suspect? - Blood draw for type/screen (Possible hemorrhage) - Continuous electronic fetal monitoring - Initiation of 2 large bore IV catheters - Assessment of peripads to assess amount of bleeding/saturated pads per hour - CORRECT ANSWER When caring for an expectant mother suspected of having placenta pre via, what nursing interventions are appropriate? Digital vaginal examinatio - CORRECT ANSWER When placenta previa, abrupt placenta & vaginal bleeding of unknown origin are suspected, what nursing intervention is completely CONTRAINIDICATED? Osteoporosis - PPI's decrease absorption of calcium and a bone density test will need to - CORRECT ANSWER For patients on long term therapy with PPI's, what is an adverse effect that may develop over time? What test will need to be performed? - Alcohol/drug intoxication - Altered consciousness - Mental illness (Hearing voices) - Chemical influence (Narcotics, benzos) - If deemed a danger to self or others - CORRECT ANSWER What are contraindications to a patient being able to sign out AMA? - Documentation on discussion of risks with client's decision & understanding of these risks - Signature of client on AMA document (Client's cannot be held against will if not willing to sign) - Should be witnessed and documented - CORRECT ANSWER For a patient wishing to sign out of the hospital APA, what must be discussed with patient & what documentation must be present? Sulfonylureas - Glyburide (long acting agent); can cause severe and prolonged hypoglycemia - CORRECT ANSWER According to the beer's criteria, what anti diabetic agents are contraindicated in older adult clients? What is an example of this medication class? - Trauma - Major surgery - Prolonged immobilization (Stroke, long travel) - Pregnancy - Oral contraceptives - Underlying malignancy - Smoking - Old age - Obesity & vericose veins - Myeloproliferative disorders - CORRECT ANSWER What are some risk factors associated with DVTs? Lateral (horizontal) violence - CORRECT ANSWER Defined as acts of aggression carried out by a co-worker against another co-worker and designed to control, diminish, or devalue a colleague - Permethrin 1% cream to the head - Removal of knits with a nit comb or by hand - Frequent vacuuming of furniture, rugs and carpets - Washing of client's bedding with hot water and dried on the hottest dryer setting - Non-washable items must be sealed in a plastic bag for 2 weeks to kill lice larva - Hairbrushes, combs and hair accessories should be soaked in boiling water for 10 minutes or lice killing products for 1 hour - Other family members will also need to be treated with a pediculicide - Household pets do not transmit human lice so they do not need to be treated - CORRECT ANSWER What is the treatment and management for periculosis capitis? Within 6 months of first tooth erupting or by their first birthday - CORRECT ANSWER When does the American Dental Association recommend that a toddler has their first visit to the dentist? 70-105 mm Hg - CORRECT ANSWER What is the normal range for mean arterial pressure (MAP)? < 60 mm Hg - CORRECT ANSWER If the MAP (Mean arterial pressure) falls below what value, the vital organs may be under perfused and can become ischemic Empty their bladder - a fresh urine specimen will be necessary if a hemolytic reaction occurs to assess for hemolyzed RBCs - CORRECT ANSWER Prior to administration of packed RBC's, what should the nurse ask the patient to do and why? - Avoid intake of fluids with meals - Consume low carb meals - Eat small, frequent meals - Eat slowly in a relaxed environment - Consume meals high in protein, fat & fiber which take longer to digest - Avoid sitting up after meals, lying down is preferred to slow gastric emptying - CORRECT ANSWER For patients that undergo a gastrectomy, what are instructions for the nurse to include to prevent dumping syndrome? Supine - to delay gastric emptying & prevent dumping syndrome - CORRECT ANSWER For patients post gastrectomy, what position is recommended after meals? Hypotension, abdominal pain, diarrhea, N&V, dizziness, generalized diaphoresis and tachycardia - CORRECT ANSWER For patients that underwent gastric surgery, what are s/s of dumping syndrome? Dumping syndrome - CORRECT ANSWER Defined as when gastric contents are emptied too rapidly into the duodenum, causing a fluid shift into the small intestine R: Rescue A: Alarm C: Contain E: Extinguish - CORRECT ANSWER Pertaining to fires in the clinical setting, what does the mnemonic RACE stand for? - Assess for intolerance to tube feedings by assessing gastric residual, abdominal dissension and bowel movements - Assess tube placement at regular intervals - Keep HOB > 30 degrees - Keep endotracheal cuff INFLATED, low cuff pressure increases risk for aspiration - Suction any present secretions - Use caution when administering sedatives - Avoid bolus tube feedings*** - CORRECT ANSWER When caring for patients on mechanical ventilation receiving tube feedings, how does the RN prevent aspiration in these clients? Bolus tube feedings - CORRECT ANSWER When caring for patients on a mechanical ventilator receiving tube feedings, what should be absolutely avoided which puts the patient at increased risk for aspiration? No less than q4hr - CORRECT ANSWER For clients on mechanical ventilation receiving tube feedings, how often should gastric residual be checked by the RN? Systolic BP + diastolic BP x2 /3 (Multiply by 2 divide by 3) - CORRECT ANSWER How do you calculate MAP (Mean arterial pressure)? - To witness that the client signed the consent voluntarily and that the client was competent at the time of signing - That the client received enough necessary information and has no remaining questions r/t procedure - Must document the time/date of the signature & when informed consent was given - CORRECT ANSWER What is the RN's role in informed consent? Transurethral resection of the prostate (TURP) - CORRECT ANSWER This procedure involves the insertion of a resectoscope to excise obstructing prostate tissue Urine remains light pink without clots - CORRECT ANSWER When caring for a patient post TURP, while receiving continuous bladder irrigation, how does the RN know that the irrigation is running at an adequate rate? Urine is dark red with positive clots present - CORRECT ANSWER When caring for a patient post TURP, while receiving continuous bladder irrigation, how does the RN know that the irrigation is NOT running at an adequate rate? Gonorrhea and chlamydia - CORRECT ANSWER What two STD's are the leading causes of pelvic inflammatory disease (PID) and infertility? - Adherence to a high calorie diet (4000-5000/day) - Consumption of 6 full meals and snacks high in protein, carbs and vitamins/minerals - Avoidance of high fiber foods do to increased stimulation of GI tract - Avoidance of stimulating substances (Caffeinated drinks, coffee, tea, soda) - Avoidance of spicy foods (Will increase gastric stimulation) - CORRECT ANSWER For patients diagnosed with hyperthyroidism, what patient education points does the nurse need to touch on? Hyperthyroidism - CORRECT ANSWER This endocrine disorder refers to sustained hyper functioning of the thyroid gland due to excessive secretion of thyroid hormones (T3, T4) leading to increased metabolic rate - Use interpreter of same gender if topic sensitive - Make good eye contact when communicating with client (Communication with client, not interpreter) - Interpreter should translate words literally - RN should use simple terms not medical terms - RN should speak slowly and in short sentences - CORRECT ANSWER When caring for a patient who does not speak their native tongue, how should the nurse implement proper use of a professional interpreter? Suctioning - CORRECT ANSWER This nursing intervention involves removing secretions from a client's airway No more than 10 seconds - CORRECT ANSWER When suctioning a patient, how long should suction be applied for? 1-2 minutes - CORRECT ANSWER When suctioning a patient, how long should the nurse wait between suction passes? Hyperoxygenate the patient with 100% o2 - CORRECT ANSWER Prior to suctioning a patient, what does the nurse need to do? Between 100-120 mm Hg for adults & 50-75 mm Hg for children - CORRECT ANSWER When suctioning a patient, at what pressure should the suction be set at for adults? Children? 150,000-400,000 - CORRECT ANSWER What is the normal lab value for platelets? - Reduce stimuli in bedroom (Reading, TV) - Avoid naps later in day - Keep bedroom slightly cool, quiet & dark - Avoid caffeine, nicotine and alcohol within 6 hours of sleep - Avoid exercising or strenuous activity within 6 hours of going to bed - Avoid going to bed hungry - Practice relaxation techniques - CORRECT ANSWER For patients with trouble sleeping, what should the RN educate the patient about for better sleep patterns? - Counseling or psychosocial support - Gentle massage with water based lotion to alleviate itching and minimize scarring - Plan for reconstructive surgery - Pressure garments to prevent hypertrophic scars and promote circulation - ROM exercises to prevent contractors - Sunscreen to prevent sunburn - CORRECT ANSWER For patients in the rehabilitation phase of burn injury, what are proper interventions to include in plan of care? - Do not eat, drink or smoke on the day of test - Avoid caffeine 24 hours before test - Avoid decaffeinated products 25 hours before test - Ask HCP about diabetes medications prior to test; Hypoglycemia can result while patient is NPO for test - Avoid nitrates, dipyridamole and BBs prior to test*** will masks s/s of angina - CORRECT ANSWER For patients set t undergo a pharmacologic nuclear stress test, what are pre-procedure client instructions? - Discomfort, redness and a few vesicles at the injection site - CORRECT ANSWER What are some s/s of varicella immunization administration? - Cover the vesicles with clothing or a small bandage until they are dried or crusted over (No longer contagious) - CORRECT ANSWER What is proper nursing intervention when a patient develops several small vesicles at the injection site of varicella vaccine? Aspirin - CORRECT ANSWER What medication is completely contraindicated in children due to the development of Reye syndrome? Positive abdominal tenderness & rigid abdomen - CORRECT ANSWER For a patient that underwent a colonoscopy, what is a concerning finding that could indicate perforation of the colon? - Keep cord stump clean and dry - Keep cord open to air to allow for adequate drying - Do NOT apply antiseptics (Alcohol, chlorhexidine) to stump - Report any s/s of infection to HCP - CORRECT ANSWER When caring for a newborns umbilical cord, what is proper technique and what needs to be reported to HCP? 2-3 days - CORRECT ANSWER How long after cutting the umbilical cord will it begin to shrivel and turn black? Lactose intolerance - CORRECT ANSWER This GI disorder is due to a deficiency in the enzyme lactase, which is required for digestion of lactose Calcium and vitamin D - CORRECT ANSWER For patients with lactose intolerance, what vitamin supplementation will be required? Hemophilia - CORRECT ANSWER This disorder is caused by a deficiency in coagulation proteins, lacking factor VIII or factor IX Joint destruction - Most frequent sites of bleeding are the joints, especially the knee, causing chronic swelling and deformity - CORRECT ANSWER For patients diagnosed with hemophilia, what is a long term complication associated with this bleeding disorder and why? Frostbite - CORRECT ANSWER This disease involves tissue freezing, resulting in ice crystal formation in intracellular spaces that causes peripheral vasoconstriction, reduced blood flow , vascular stasis and cell damage - Remove clothing and jewelry to prevent constriction - DO NOT MASSAGE, rub, or squeeze area involved - Immerse affected area in heated water - Provide analgesia as appropriate - Elevate injured areas due to possible edema - Keep wounds open and allow them to dry before applying a loose, sterile dressing - CORRECT ANSWER What is proper treatment of frostbite? Serotonin syndrome - CORRECT ANSWER Defined as a potentially life threatening condition that develops when drugs affecting the body's serotonin levels are administered simultaneously in overdose St. John's Wort - CORRECT ANSWER What herbal supplement taken in conjunction with an antidepressant can potentially cause serotonin syndrome? - Mental status changes (Disorientation, anxiety) - Autonomic dysregulation (Hyperthermia, diaphoresis, tachycardia, HTN) - Neuromuscular hyperactivity (Tremor, muscle rigiditu, clonus, hyperreflexia) - Fever - Warm, moist skin - CORRECT ANSWER What are examples of clinical manifestations that may be present with serotonin syndrome? 2nd ICS right sternal border - CORRECT ANSWER When auscultating heart sounds, where is the aortic valve located? 2nd ICD left sternal border - CORRECT ANSWER When auscultating heart sounds, where is the pulmonic valve located? 3rd ICS left sternal border - CORRECT ANSWER When auscultating heart sounds, where is the erg's point located? 5th ICS lower left sternal border - CORRECT ANSWER When auscultating heart sounds, where is the tricuspid valve located? 5th ICS midclavicular line - CORRECT ANSWER When auscultating heart sounds, where is the mitral valve located? Mitral valve - CORRECT ANSWER This valve is also called the apex & point of maximum impulse 5th ICS midclavicular line - CORRECT ANSWER Where is the apex or the point of maximum impulse located for auscultation? - Gi bicarb losses (Diarrhea) - Ketoacidosis (DKA, alcoholism, starvation) - Lactic acidosis (Sepsis, hypoperfusion) - Renal failure (Hemodialysis with inaccessible AV fistula) - Salicylate toxicity - CORRECT ANSWER What are common causes of metabolic acidosis? - CNS disruption (Stroke, infection, trauma, neurosurgery) - Malignancies (Small cell lung carcinoma) - Drugs (Desmopressin, carbamazepine) - Pulmonary disorders (Pneumonia) - CORRECT ANSWER What are examples of causes of SIADH? SIADH - Syndrome of inappropriate antidiuretic hormone - CORRECT ANSWER Defined as an endocrine disorder in which too much ADH is produced causing water retention, increased total body water and dilution serum hyponatremia q48hr with gauze dressing or q7days with a transparent semipermeable dressing - CORRECT ANSWER How often should the dressing on a central line (PICC)be changed? ALL infusing medications (except vasopressors) must be paused before drawing blood to prevent false interpretation of the client's levels - CORRECT ANSWER When drawing blood from a PICC line, what other infusing medications should be paused and why? 10-15 seconds - CORRECT ANSWER How long should an antiseptic be used to "scrub the hub" of a central venous catheter before medication administration? Alcohol or chlorhexidine/alcohol - CORRECT ANSWER What antiseptic solution is used to "scrub the hub" of a central venous catheter before medication administration? - Tachycardia - Dysrhythmias - Myocardial ischemia - CORRECT ANSWER What are examples of adverse effects of the sympathomimetic inotropic medication, dopamine? Dopamine is a sympathomimetic inotropic medication used to improve hemodynamic status in clients with shock and HF - Enhances cardiac output by increasing myocardial contractility, increasing HR and elevating blood pressure through vasoconstriction - CORRECT ANSWER What class of medication is dopamine in, MOA and are the therapeutic effects of administration? 2-8 mm Hg - CORRECT ANSWER What is the range for normal central venous pressure (CVP)? 800-1200 dynes/sec/cm5 - CORRECT ANSWER What is the range for normal systemic vascular resistance? A non biologic disease-modifying antirheymatic drug (DMARD) used to treat RA - CORRECT ANSWER What kind of drug is Methotrexate and what is it used to treat? - Bone marrow suppression - THROMBOCYTOPENIA** - Hepatotoxicity - Immunosuppression - GI irritation (N&V, diarrhea) - CORRECT ANSWER What are some AE associated with Methotrexate administration? Thrombocytopenia - CORRECT ANSWER For patients taking Methotrexate, what AE is suspected if petechiae is assessed on a patients body? Precipitous birth - CORRECT ANSWER Defined as when a newborn is delivered < 3 hours after the onset of contractions Placental separation & expulsion - CORRECT ANSWER Signs that this is happening during the birth process include, cord lengthening, a gush of blood, uterine cramping and vaginal pressure Fundal massage - CORRECT ANSWER What nursing intervention is performed after expulsion of the placenta to increase uterine tone and decrease bleeding? To increase uterine tone and decrease risk for post partum hemorrhage - CORRECT ANSWER Why is fundal massage performed? Peritonitis - CORRECT ANSWER Defined as a common but serious complication of peritoneal dialysis Cloudy peritoneal effluent - CORRECT ANSWER What is the typical, earliest indication of peritonitis infection? - Low grade fever - Chills - Generalized abdominal pain - Rebound tenderness - Board like abdomen* - CORRECT ANSWER What are late clinical manifestations of peritonitis? - Identify patient using 2 identifiers - Perform hand hygiene and apply clean gloves - Elevated HOB > 30 degrees and keep elevated for at least 30 minutes to prevent aspiration - Validate tube placement by checking gastric pH and comparing it with measurement of tube length at time of insertion - Check residual volume - Flush tube with 30 mL of water - Administer prescribed enteral feeding solution by setting the rate on infusion pump - CORRECT ANSWER What are the steps for administering a continuous enteral feeding to a patient? Asians - CORRECT ANSWER What ethnic group is at highest risk for developing a dry cough with ACE inhibitors? - Elevating the affected harm to heart level (Reduces fluid retention and lymphedema) - Hand and arm exercises (to maintain muscle tone, prevent contractors and improve circulation) - Keeping patient in semi fowler position - Place sign over bed that says "restricted left/right extremity" - No BP, venipuncture or injections on affected extremity - CORRECT ANSWER What are vital nursing interventions when caring for a patient post mastectomy? - Pneumococcal vaccine & annual influenza vaccine - CORRECT ANSWER What is the current vaccination recommendations for patients with COPD, regardless of age? Flail chest - multiple ribs sustained multiple fractures and become independent of chest wall, floating on top of the lung and pleura - CORRECT ANSWER When triaging victims after a major disaster, what does see saw movement with respirations indicate? Umbilical cord prolapse - CORRECT ANSWER Complication of pregnancy that occurs when the umbilical cord slips below the presenting fetal part and may cause cord compression and impaired fetal oxygenation Emergency c-section unless vaginal birth is imminent and considered safe by HCP - CORRECT ANSWER What is the treatment for umbilical cord prolapse? Position the client on hands and knees to relieve cord compression - CORRECT ANSWER Upon vaginal assessment of a patient in labor, the nurse visualizes a loop of umbilical cord protruding from the vagina. What is the nurse's first action? - Self-induced vomiting - Fasting - Laxative abuse - Excessive exercise - CORRECT ANSWER What are examples of weight maintenance behaviors of a patient diagnosed with bulimia nervosa? 1-2 hours after meals - CORRECT ANSWER For clients diagnosed with bulimia nervosa, when is it vital the nurse monitor the patient? Infiltration - CORRECT ANSWER Complication of IV therapy defined as leaking of solution into surrounding tissues of the infusion site - D/C IV line ASAP - Start IV on opposite extremity - Elevate the affected extremity to decrease swelling - Apply a cold/warm moist compress to site - Notify HCP if other complications develop - CORRECT ANSWER What are proper nursing interventions when observing that an IV line is infiltrating? q82-96hr - CORRECT ANSWER How often should peripheral IV sites be changed? Abdominal aneurysm - CORRECT ANSWER Upon abdominal assessment of a patient, the nurse notes a pulsatile mass in the periumbilical area. What does she suspect? - Back/abdominal pain (Due to compression of nearby anatomical sites) & a pulsatile abdominal mass - CORRECT ANSWER What are the clinical manifestations of a AAA? - Emergency surgery to repair the aneurysm to prevent rupture - CORRECT ANSWER What is the treatment for a AAA? Nystatin - CORRECT ANSWER What is the preferred treatment for oral candidiasis due to corticosteroid/antibiotic use? Swish liquid in mouth for several minutes and then swallow it - CORRECT ANSWER For the treatment of oral candidiasis, how should the nurse instruct the patient to administer oral nystatin? Gloves - the most dirty - CORRECT ANSWER When removing PPE after a wound dressing change for a patient with MRSA, what article of PPE should be removed first? Pericardial effusion - CORRECT ANSWER Defined as a buildup of fluid in the pericardium Cardiac tamponade - CORRECT ANSWER Defined as a serious complication of a pericardial effusion Emergency pericardiocentesis - Needle inserted into pericardial sac to remove excess fluid constricting the heart from beating - CORRECT ANSWER What is the immediate treatment of cardiac tamponade? - Hypotension - Narrowed pulse pressure - Muffled/distant heart tones - JVD - Pulsus paradoxus - Dyspnea, tachypnea - Tachycardia - Weak, thready pulses - CORRECT ANSWER What are the s/s and clinical manifestations of cardiac tamponade? 14 days - to prevent risk of developing serotonin syndrome - CORRECT ANSWER How long after cessation of an MAOI can an SSRI be started and why? General anesthesia & a skeletal muscle relaxant - CORRECT ANSWER What medications are administered prior to ECT therapy? Confusion and memory loss - CORRECT ANSWER What are common side effects after ECT therapy? Anticonvulsants - ECT is used to provoke convulsion - CORRECT ANSWER What medication class is contraindicated prior to ECT therapy and why? Hypertensive crisis - CORRECT ANSWER Defined as a life threatening emergency & if left untreated will cause intracranial hemorrhage, HF, MI, renal failure, aortic dissection or retinopathy - IV vasodilators (Nitroprusside sodium) - CORRECT ANSWER What is the emergency treatment for hypertensive crisis? - To decrease the MAP by no more than 25% or to maintain MAP at 110-115 mm Hg - CORRECT ANSWER When caring for a patient diagnosed with a hypertensive crisis, what is the initial goal of treatment? Too rapid of a drop may cause decreased perfusion to the brain, heart and kidneys, resulting in stroke, renal failure or MI - CORRECT ANSWER When caring for a patient diagnosed with a hypertensive crisis, what is the risk in lowering the BP too quickly? Live vaccinations - IVIG administration will interfere with desired immune response to live vaccines (Varicella, MMR, influenza) - CORRECT ANSWER After IVIG administration for Kawasaki disease, what immunizations should be delayed in this client? Hypocalcemia - due to chronic ingestion of alcohol - CORRECT ANSWER What is a probable electrolyte imbalance seen in chronic pancreatitis and why? - Numbness/tingling in finers and lips - Trousseau's sign (BP cuff) - Chvostek's sign (Facial twitching) - CORRECT ANSWER What are s/s of hypocalcemia? CVP - Central venous pressure - CORRECT ANSWER In critical care, this is a measurement of right ventricular preload and reflects fluid volume status Right ventricular failure of FVE - CORRECT ANSWER What does a high CVP potentially indicate? Hypovolemia (FVD) - CORRECT ANSWER What does a low CVP potentially indicate? Borderline personality disorder (BPD) - CORRECT ANSWER Patients diagnosed with this mental illness often make suicidal threats, gestures and attempts & may use these behaviors when real/perceived abandonment is present Buck's traction - CORRECT ANSWER This type of traction is used to immobilize a fractured hip and reduce pain and swelling until the client can be stabilized for surgery Skin/neurovascular assessment q2-4hr - CORRECT ANSWER When caring for patients in Buck's traction, what is the MOST important nursing assessment? Supine in the center of the bed - CORRECT ANSWER For patients in Buck's traction, how are they positioned in bed? Hanging freely and not touching the bed/floor - CORRECT ANSWER For patients in Buck's traction, how should the weights be visualized? - Peripheral pulses - Capillary refill - Color/temperature of extremities - Sensation - Movement - 5 P's (Pain, pallor, pulselessness, paresthesia, paralysis) - CORRECT ANSWER What are the components of an adequate neuromuscular nursing assessment? Above the level of the heart - CORRECT ANSWER How should the extremity of a patient in Buck's traction be kept? From side-to-side - CORRECT ANSWER When repositioning a patient in Buck's traction, how should they NOT be turned? Abduction pillow - CORRECT ANSWER For patients in Buck's traction, what is used to prevent adduction of the hips when repositioning? LFT's - CORRECT ANSWER Prior to beginning statin treatment, what lab value does the nurse need to check? REPORT muscle aches/pain/weakness!! - CORRECT ANSWER What is an important education point prior to beginning a patient on statin therapy? - Avoid spicy, acidic foods - Avoid substances that stimulate acid secretion (NSAIDs, alcohol, caffeine, chocolate, tobacco) - Reduce stress and obtain sufficient rest - CORRECT ANSWER What are lifestyle modifications that need to be implemented if diagnosed with peptic ulcer disease (PUD)? - GI Bleeding: Report orthostatic hypotension, tachycardia & melena/black stools - Perforation: Report increased epigastric pain, N&V, fever - CORRECT ANSWER For patients diagnosed with peptic ulcer disease (PUD), what should the nurse educate the pt about to report to the provider? Infection with h. pylori and treatment with NSAIDs - CORRECT ANSWER What are two risk factors for developing peptic ulcer disease? Antibiotics for infection and PPIs for acid suppression - CORRECT ANSWER What is the treatment for peptic ulcer disease? Aspiration due to movement of placement of NG tube - CORRECT ANSWER If a patient achieving enteral tube feedings via NG tube begins to experience dyspnea and a productive cough, what does the nurse suspect has occurred? How? Stop tube feeding ASAP and assess tube placement (Tube insertion depth, gastric pH) - CORRECT ANSWER If a patient achieving enteral tube feedings via NG tube is suspected of having aspirated, what is the nurses first action? Must be taken within 5 days of intercourse; Efficacy is reduced after 72 hours (3 days) - CORRECT ANSWER How long should emergency contraception be taken after unprotected sex? When is efficacy reduced? 7 days - CORRECT ANSWER For a patient recently beginning oral contraception, for what amount of time does the nurse need to educate the pt on how to use back up contraception? Diabetes insipidus (DI) - CORRECT ANSWER Defined as condition that occurs due to insufficient production/suppression go antidiuretic hormone (ADH) - After manipulation of the pituitary - Brain tumors - Head injury - CNS infections - CORRECT ANSWER When can DI potentially develop? Diabetes insipidus (DI) - CORRECT ANSWER This disorder is characterized by polyuria, polydipsia and decreased urine specific gravity Desmopressin (DDAVP) - CORRECT ANSWER What is the preferred treatment of DI? Severe diaper rash - Due to catheterization insertion site being in the groin, this could lead to a bloodstream infection - CORRECT ANSWER For infants undergoing balloon angioplasty to repair congenital pulmonic stenosis, what assessment finding would potentially delay this procedure? - Encourage the patient to cut nails short - Wear cotton gloves - Wear long sleeved shirts to avoid injury by itching - Taking baking soda baths - Applying calamine lotion - Applying cool, wet cloths to irritated skin - CORRECT ANSWER For patients diagnosed with cirrhosis, what are nursing interventions to reduce pruritus? Due to buildup of bile salts beneath the skin - CORRECT ANSWER For patients diagnosed with cirrhosis, why do they experience pruritus? Cholestyramine (Questran) - CORRECT ANSWER What medication will be prescribed for patients with cirrhosis to decrease the pruritus experienced due to accumulation of bile salts underneath the skin? - Powder; Must be mixed with food (Applesauce) or juice and should be given one hour AFTER other medications are administered - CORRECT ANSWER What is proper medication administration for Cholestyramine (Questran) when treating pruritus in cirrhosis? Kawasaki Disease (KD) - CORRECT ANSWER This childhood disorder is also known as mucocutaneous lymph node syndrome Kawasaki Disease (KD) - CORRECT ANSWER This pediatric disease is characterized by > 5 days of fever, bilateral nonexudative conjunctivitis, mucositis, cervical lymphadenopathy, rash and extremity swelling To prevent coronary aneurysms and subsequent occlusion - CORRECT ANSWER Why is IV immunoglobulin and aspirin given to treat Kawasaki Disease (KD)? To prevent heart disease - CORRECT ANSWER What is the therapeutic effect of IVIG and aspirin for Kawasaki Disease (KD)? - Patient should take potassium tablets with plenty of water - Patient should remain sitting up right for > 30 minutes after ingestion - CORRECT ANSWER For patients prescribed loop diuretics, how should potassium supplementation be taken? Pill induced esophagitis and erosion of esophagus - CORRECT ANSWER If potassium supplementation tablets are not taken with enough water or if the patient does not sit upright for > 30 minutes after ingestion, what can potentially happen? Vitamin D - CORRECT ANSWER What vitamin helps with calcium absorption? - Apply clean gloves and position the client supine with knees and feet raised - Lubricate the tip of suppository with water soluble jelly - Insert suppository past internal sphincter using the fifth finger - Angle suppository and guide it along the rectal wall - Hold buttocks together for several minutes or until urge to defecate has passed - CORRECT ANSWER When administering a rectal suppository to an infant, what is proper procedure? Retinoblastoma - CORRECT ANSWER Defined as a unilateral or bilateral retinal tumor and is the most common childhood intraocular malignancy Parents report a white "glow" of the pupil - CORRECT ANSWER How is retinoblastoma usually first recognized in children? Leukocoria (white glow of pupil) & strabismus (Misalignment of eyes) - CORRECT ANSWER What are the most common clinical manifestations of retinoblastoma? Radiation therapy or enucleation (Removal of eye) - CORRECT ANSWER What is the treatment for retinoblastoma in children? - Frequent use of pacifier - Infant's immunization status - Infant's position while drinking from bottle - CORRECT ANSWER What are risk factors for infants developing recurrent otitis media? Otitis media (OM) - CORRECT ANSWER Defined as inflammation or infection of the middle ear - Recurring exposure to tobacco spoke - Regular pacifier use - Drinking from a bottle when lying down - Lack of immunizations - CORRECT ANSWER What are modifiable risk factors to alter to prevent recurrent otitis media in infants? Bathing or swimming and getting water in the ears - CORRECT ANSWER What is a risk factor for infants and children contracting otitis externa? - Cardiac pacemaker - Implantable cardioverter defibrillator (ICD) - Cochlear implant - Retained metallic foreign body, such as in the eye - CORRECT ANSWER What are absolute contraindications for an MRI? Sucralfate - CORRECT ANSWER This medication is an oral medication that forms a protective layer in the GI mucosa which provides a physical barrier against stomach acids and enzymes - 1 hour prior to meals on an empty stomach with a full glass of water - Provides protective layer and physical barrier against stomach acids and enzymes - CORRECT ANSWER When is sucralfate typically prescribed and why? Morphine - CORRECT ANSWER What medication is typically used to manage dyspnea, tachycardia and restless associated with withdrawing mechanical ventilator support at end of life? Bladder exstrophy - CORRECT ANSWER Defined as a congenital disorder in which the bladder was not developed in the abdominal cavity during fusion in the embryo and is exposed externally at birth Placing a protective film of plastic over the exposed bladder to keep the tissue moist and help prevent infection - CORRECT ANSWER What is the most important nursing intervention when caring for an infant with bladder exstrophy? Autonomic dysreflexia - CORRECT ANSWER Defined as a life threatening condition caused by sensory stimulation that occurs in clients who have a spinal cord injury at T6 or higher - Asparagus - Turnip/mustard greens - Fortified breakfast cereal - Cooked dried beans - Liver - Leafy greens (Broccoli, spinach) - Beats - Rice - Peanut butter - CORRECT ANSWER What are examples of foods high in folic acid? - Mild: Anxiety, insomnia, diaphoresis, palpitations, GI upset Seizures: Single or generalized Hallucinosis: Visual, auditory or tactile hallucinations - CORRECT ANSWER What are some s/s of alcohol withdrawal syndrome (DTs)? - Confusion - Agitation - Fever - Tachycardia - Hypertension - Diaphoresis - Hallucinations - CORRECT ANSWER What are s/s of delirium tremens (DTs)? Administration of benzos - CORRECT ANSWER How can the DT's be prevented during hospitalization? The nurse should place her fingers halfway between the shoulder and elbow, palpating the brachial artery - CORRECT ANSWER When palpating a pulse on an infant, where should the nurse place her fingers? What artery is this? Ringworm aka tinea corporis - CORRECT ANSWER Defined as a fungal infection on the superficial keratin layers of the skin, hair and/or nails - Teaching appropriate hygiene (Hand washing) - Limited contact with infected person's personal items (Hair brush) - Treatment with prescribed shampoo and/or topical/oral medications - CORRECT ANSWER What are examples of proper management of ringworm? Balloon tamponade tube - CORRECT ANSWER This intervention is used to temporarily control bleeding from esophageal varicose IV calcium gluconate - CORRECT ANSWER For a symptomatic patient with a potassium level of 7.3, what is the fastest intervention that can be implemented to decrease dysrhythmias associated with hyperkalemia? C - Calcium gluconate B - Bicarbonate I - Insulin (Regular) G - Glucose (D20W) K - Kayexalate D - Diuretics ROP - Removal of patient (Dialysis) - CORRECT ANSWER What does CBIG KDROP stand for related to hyperkalemia? Remove any tubes or IV lines - CORRECT ANSWER When an autopsy is REQUIRED due to legal reasons, what should the nurse NOT do during post portum care? Cushing syndrome - CORRECT ANSWER This disorder is the result of prolonged exposure to excess corticosteroids, especially glucocorticoids - Acne - Hirsutism (Unwanted hair growth) - Menstrual irregularities - Truncal obesity - Hypertension - Moon face - Buffalo hump - Hyperglycemia - Easy bruising - Muscle weakness and osteoporosis - CORRECT ANSWER What are the clinical manifestations associated with Cushing syndrome? - Hyperglycemia - Hypernatremia - Hypokalemia - CORRECT ANSWER What electrolyte abnormalities are present in Cushing syndrome? - Hypoglycemia - Hyponatremia - Hyperkalemia - CORRECT ANSWER What electrolyte imbalances are present in Addison's disease? Breastfeeding - HIV can be transmitted to the newborn via breast milk - CORRECT ANSWER What is contraindicated for HIV positive mothers post delivery? Hospice - CORRECT ANSWER Defined as a program of care and support that assists clients who have a terminal illness by improving their quality of life through comfort measures - Must be terminally ill with a life expectancy of 6 months or less - Willing to accept palliative care - CORRECT ANSWER What is the eligibility for hospice? - Hospice can be provided in the home, nursing home or in the hospital - Hospice care is covered by Medicare - Hospice helps people die naturally in their own time - If your condition improves, you can be discharged from hospice - CORRECT ANSWER What are appropriate statements for the nurse to make when discussing hospice care with a patient and their family? 10-20 mcg/mL - CORRECT ANSWER What is the therapeutic drug level for Phenytoin (Dilantin)? - Horizontal nystagmus - Gait unsteadiness - Slurred speech - Lethargy - Confusion - Coma - CORRECT ANSWER What are early signs of Phenytoin (Dilantin) toxicity? NSAIDs - CORRECT ANSWER What OTC medications are contraindicated during pregnancy? Magnesium sulfate - CORRECT ANSWER This medication is a central nervous system depressant that is commonly prescribed to prevent seizures in clients with preeclampsia - Continuously assess deep tendon reflexes - Have calcium gluconate at bedside (Reversal agent) - Decrease environmental stimuli to minimize risk for seizure activity - Decrease lights in room and other environmental stimuli - CORRECT ANSWER For patients receiving magnesium sulfate for preeclampsia, what are proper nursing assessments/interventions to perform at this time? Calcium gluconate - CORRECT ANSWER What is the reversal agent for magnesium sulfate? - Administration of stool softeners (Colace) to prevent straining - Administering antiemetics (Zofran, Reglan) to prevent N&V which will increase intra abdominal pressure - Applying an abdominal binder to support the incision - Monitoring BG to prevent infection and promote wound healing - Educating about splinting the abdomen by holding a pillow or folded blanket against abdomen when coughing and moving - CORRECT ANSWER What nursing interventions are vital to prevent the occurrence of dehiscence after abdominal surgery? - Keep bottles/nipples/caps as clean as possible - Wash the tops of formula cans prior to opening to prevent contamination - Prepared formula must be kept in fridge and discarded after 48 hours - Prepared bottles can be warmed in a pain of hot water NOT MICROWAVE - Test temperature of bottle on inner wrist before giving to infant - CORRECT ANSWER What is proper storage/handling of formula/milk related to infant feeding? Microwaving formula can cause irregular heating and mouth burns in infants - CORRECT ANSWER Why is infant formula NEVER microwaved? - Water intoxication r/t hyponatremia - CORRECT ANSWER Why should formula never be diluted or concentrated? Addison's disease - CORRECT ANSWER Defined as primary adrenocortical insufficiency or hypo function of the adrenal cortex - Glucocirticoids - Androgens - Mineralocorticoids - CORRECT ANSWER What are the adrenal glands responsible for secreting? - Bronze pigmentation of skin in sun exposed areas - Vitiligo (Patchy/blotchy skin) - Slow, progressive onset of weakness and fatigue - Anorexia/weight loss - Orthostatic hypotension - Hyponatremia & Hyperkalemia - Salt cravings - N&V - Depression and irriaibility - CORRECT ANSWER What are clinical manifestations of Addison's disease? Epstein-Barr virus - CORRECT ANSWER Infectious mononucleosis (Mono) is caused by what virus? Spleen rupture - CORRECT ANSWER What is a serious complication of infectious mononucleosis? Lithium - CORRECT ANSWER This medication is a mood stabilizer that is most often used to treat bipolar affective disorders 0.6-1.2 - CORRECT ANSWER What is the therapeutic range of Lithium? (Think Cr) 0.6-1.2 - CORRECT ANSWER What is the therapeutic range of Lithium? - Dehydration - Decreased renal function - Diet low in sodium - Drug-drug interactions (NSAIDs, thiazide diuretics) - CORRECT ANSWER What are some risk factors for developing lithium toxicity? Sodium - CORRECT ANSWER Reduction in which electrolyte potentiates lithium toxicity? - Verify unresponsiveness by tapping/shaking & saying "Hello are you all right?" - Activate emergency response system by calling for help or calling 911 - Simultaneously check for the carotid pulse and assess for spontaneous breathing for no more than 10 seconds - Attempt CPR if no pulse is felt - CORRECT ANSWER What is the sequence of basic life support (BLS) for an unconscious, pulseless patient? 100-120/min - CORRECT ANSWER At what rate should chest compressions be in BLS? Elevated bilirubin & elevated reticulocytes - CORRECT ANSWER During a vaso-occlusive crisis, a complication of sickle cell anemia, what is there an increased presence of in the laboratory values? Addisonian crisis - life threatening complication - CORRECT ANSWER For a patient diagnosed with Addison's disease, what can abrupt discontinuation of corticosteroids lead to? Cataracts - CORRECT ANSWER What is an ocular problem that results from long term corticosteroid use, specifically glucocorticoids? True contractions - CORRECT ANSWER These types of contractions come in regular intervals; frequency, duration and intensity increase over time Braxton Hicks Contractions - CORRECT ANSWER These types of contractions come in irregular intervals; No increase in frequency, duration or intensity & may dissipate over time True contractions - CORRECT ANSWER These types of contractions increase despite implementation of comfort measures (Change of position, hydration, walking) Braxton Hicks Contractions - CORRECT ANSWER These types of contractions may lessen or dissipate with comfort measures (Walking, change of position, hydration) True contractions - CORRECT ANSWER These types of contractions cause an increase in cervical dilation and effacement Braxton Hicks Contractions - CORRECT ANSWER These types of contractions cause no cervical change in the mother Sudden infant death syndrome (SIDS) - CORRECT ANSWER Defined as the leading cause of death among infants aged 1 month to 1 year - Place infants less than 1 year old on their BACK (Supine) to sleep - Firm mattress should be chosen - Avoid soft objects such as stuffed animals, heavy blankets etc - Avoid bumper crib pads* - Maintain SMOKE FREE house hold - Avoid overheating of infant* - Encourage use of pacifier - Breast feeding is encouraged - Keept infant up to date with immunizations - CORRECT ANSWER What measures does the RN need to educate new parents about r/t prevention of SIDS? Breastfeeding & up to date immunizations - CORRECT ANSWER What two factors contribute greatly in the prevention of SIDS? PEEP - Positive end-expiratory pressure - CORRECT ANSWER This ventilator setting applies a given pressure at the end of expiration during mechanical ventilation It counteracts small airway collapse and keeps alveoli open so that they can participate in gas exchange - CORRECT ANSWER What is the therapeutic effect of PEEP? 5 cm of h2o - CORRECT ANSWER At what level is PEEP typically kept on? 10-20 cm of h2o - CORRECT ANSWER What is the range for high levels of PEEP? Overdistension and rupture of alveoli resulting in barotrauma to the lung - CORRECT ANSWER What can continuously high levels of PEEP cause? - Verify informed consent - Gather the lumbar puncture tray and needed supplies - Explain the procedure to older child and adult - Have client empty their bladder - Place client in appropriate position (Side lying with knees drawn up to chest) - Assist client in maintaining proper position - Provide a distraction measure throughout procedure 0 Label specimen containers as they are collected - Apply bandage to the insertion site - Deliver specimens to lab - CORRECT ANSWER What is the nurses role when assisting with an LP? Toxoplasmosis - CORRECT ANSWER Defined as a parasitic infection which may be acquired from exposure to infected cat feces or ingestion of undercooked meat or soil contaminated fruits/veggies Lateral recumbent or sitting up right - CORRECT ANSWER What position is a patient typically placed in for an LP? - Patient must lie flat with no pillow for at least 4 hours to reduce chance of CSF leaking resulting in a headache - Increase fluid intake for at least 24 hours to prevent dehydration - CORRECT ANSWER After an LP, what does the nurse need to educate the patient on related to positioning? Placental insufficiency - CORRECT ANSWER What do late decelerations indicate? - Administer supplemental o2 via facemask - Reposition patient to right or left side - STOP oxytocin infusion - Administer IV fluid bolus - CORRECT ANSWER If late decelerations are seen on the fetal heart monitor tracing, what are proper nursing interventions at this time? N&V and lethargy - CORRECT ANSWER What are early signs of methadone toxicity? Delusion of Grandeur - CORRECT ANSWER What kind of delusion is this: "I need to get to Washington for my meeting with the president." Delusion of Control - CORRECT ANSWER What kind of delusion is this: "Don;t drink the tap water. That's how the government controls us." Delusion of Nihilism - CORRECT ANSWER What kind of delusion is this: " It doesn't matter if I take my medicine. I am already dead." Delusion of Somatism - CORRECT ANSWER What kind of delusion is this: "The doctor said I am fine, but I really have lung cancer." Tactile hallucination - CORRECT ANSWER What kind of hallucination is this: "I need for you to get rid of these bugs that are crawling under my skin." Auditory hallucination - CORRECT ANSWER What kind of hallucination is this: "Hear that? She told me to kill my father." Persecutory (Paranoid) delusion - CORRECT ANSWER What type of delusion is this: "Those martians are trying to poison me with that tap water." - High fever - Ear pain - Irritability/restlessness - Loss of appetite - Pulling on affected ear - Bulging/red tympanic membrane - CORRECT ANSWER What are clinical manifestations of acute otitis media (AOM)? Red tag - CORRECT ANSWER During disaster triage, these patients have suffered life-threatening injuries with good prognosis after minimal intervention Yellow tag - CORRECT ANSWER During disaster triage, these patients have suffered injuries requiring treatment within hours Green tag - CORRECT ANSWER During disaster triage, these patients have suffered injuries requiring treatment within a few days - CORRECT ANSWER During disaster triage, these patients have suffered extensive injuries with poor prognosis regardless of treatment Vaginal hematoma - CORRECT ANSWER This complication of pregnancy occurs when trauma to the tissues of the perineum occurs during delivery Vaginal hematoma - CORRECT ANSWER A client that just delivered a term infant reports persistent, severe vaginal pain & a feeling of fullness. What does the nurse suspect? Phlebitis - CORRECT ANSWER Defined as inflammation of a vein - Pain - Swelling - Warmth at the site - Redness extending along the vein - CORRECT ANSWER What are common manifestations of phlebitis? Immediately remove the IV catheter - CORRECT ANSWER What is proper nursing intervention when assessing phlebitis? - Antihypertensives - Antipsychotics - Antidepressants - Diuretics - Vasodilators - Narcotics - CORRECT ANSWER What drug classes are commonly associated with orthostatic hypotension? - Alternate periods of rest and activity - AVOID ALCOHOL - Encourage small frequent meals - Provide oral care and avoid extremes in food temperature - Drink adequate amounts of fluids - CORRECT ANSWER What are proper nursing interventions for the acute phase of hepatitis? - Step behind client and place arms under axilla/around waist - Place feet wide apart with knees bent - Place one foot behind the other and extend front leg - Let patient slide down the extended leg to the floor - CORRECT ANSWER To prevent injury in a client who is actively falling, what are proper actions by the nurse? Anencephaly - CORRECT ANSWER This condition is considered a severe neural tube defect resulting in little to no brain tissue or skull formation in utero Many are born stillborn and those that are born alive are not compatible with life - CORRECT ANSWER What is the prognosis for infants born with anencephaly? Dialysis disequilibrium syndrome (DDS) - CORRECT ANSWER Defined as a rare but potentially life threatening complication that can occur in clients during the initial stage of hemodialysis - N&V - Headache - Restlessness - Change in mentation - Seizure activity - CORRECT ANSWER What are the clinical manifestations of dialysis disequilibrium syndrome? Contact the HCP ASAP! EMERGENCY* - CORRECT ANSWER If dialysis disequilibrium syndrome (DDS) is suspected, what is the nurses first action? Antihypertensives - CORRECT ANSWER What medication class is held prior to dialysis to minimize the risk for hypotension? Enoxaparin (Lovenox) - CORRECT ANSWER Defined as low molecular weight heparin Subq injection at a 90 degree angle, to the right or left side of abdomen (Love handles), at least 2 inches away from the umbilicus - CORRECT ANSWER How should the nurse administer LMWH, Enoxaparin (Lovenox)? Hyperkalemia & fluid volume overload (FVE) - CORRECT ANSWER For patients diagnosed with chronic kidney disease, what are they at greatest risk for? > 100 mL/hr - CORRECT ANSWER Greater than what amount of chest drainage needs to be reported to the HCP? - Report any s/s of infection (Redness, swelling, drainage, fever) - Keep a pacemaker ID card with you - Wear a medic alert bracelet - Microwave ovens are safe to use* - Report if pulse is under HCP recommendation - Do not place cell phone directly over pacemaker - AVOID MRI SCANS - Avoid lifting arm above the shoulder until approved by HCP - Air travel is not restricted - Notify airport security that you have a pacemaker (Will set off metal detector) - CORRECT ANSWER For patients that underwent placement of a permanent pacemaker, what education should the nurse give the patients? Coup- countercoup - CORRECT ANSWER This type of head injury occurs when a body in motion stops suddenly, causing contusions of brain tissue as the brain moves back and fourth within the skull Frontal lobe - Deficit in memory, speech & executive function (Decision making) Occipital lobe - Deficit in vision and interpretation - CORRECT ANSWER With a coup-cointrecoup head injury due to a car accident, what types of deficit can the nurse expect? Dabigatran (Pradaxa) - CORRECT ANSWER This medication is an example of a thrombin inhibitor that reduces the risk of clot formation and stroke in clients with chronic atrial fibrillation Capsules should be kept in their original container blister pack until time of use to prevent moisture contamination - CORRECT ANSWER For patients prescribed Dabigatran (Pradaxa), how should this medication be stored? Rifampin and rifampin derivatives reduce efficacy of oral contraceptives by increasing their metabolism; Alternative BC method will need to be implemented during treatment - CORRECT ANSWER For patients prescribed Rifapetine, a derivative of rifampin, how should the nurse instruct these patients on use of birth control? < 3-4.5 hours - CORRECT ANSWER What is the appropriate time range from TPA to treat an ischemic stroke? Myasthenia gravis - CORRECT ANSWER Defined as an autoimmune disease involving a decreased number of acetylcholine receptors at the neuromuscular junction - Weakness of skeletal muscles - Ptosis/diplopia - Bulbar signs (Difficulty speaking/swallowing) - Difficulty breathing - CORRECT ANSWER What are examples of clinical manifestations of myasthenia gravis? Anticholinesterase drugs - Pyridostigmine (Mestinon) - CORRECT ANSWER What is the treatment for myasthenia gravis? Before meals - CORRECT ANSWER In treating myasthenia gravis, when are anti cholinesterase medications administered? Aspiration - CORRECT ANSWER What is the greatest risk for patients with esophageal atresia (EA) and tracheoesophageal fistula (TEF)? - Maintaining NPO status - Positioning pt supine with HOB at > 30 degrees - Keep suction equipment at bedside - Prevent aspiration - CORRECT ANSWER What are priority nursing interventions for patients diagnosed with EA/TEF? Cognitive behavioral therapy (CBT) - CORRECT ANSWER This form of therapy can be effective in treating anxiety disorders, eating disorders, depressive disorders and medical conditions such as insomnia and smoking - Education about client's specific disorder - Self observation and monitoring - Physical control strategies - Cognitive restructuring - Behavioral strategies - CORRECT ANSWER What are the 5 basic components of cognitive behavioral therapy (CBT)? - Avoid oil based lubricants on nostrils (Flammable) - Water-soluble lubricants are acceptable - Keep oxygen canisters at least 5-10 feet away from fire sources - Do NOT increase o2 flow rate without HCP prescription; Drive to breath can be potentially decreased - Avoid nail polish remover with oxygen (Flammable) - Avoid synthetic and wool fabrics (Flammable) - Cotton blankets/fabrics are recommended - CORRECT ANSWER When discharging a patient with COPD with home oxygen, what are nursing education points that should be voiced at this time? Peak flow meter - CORRECT ANSWER Defined as a hand held device that is used to measure peak expiratory flow rate (PEFR) and is most helpful for patients with moderate-severe asthma - Move the indicator on the numbered scale to 0 before using device - Exhale as quickly and forcibly as possible through the mouthpiece - Mouthpiece evaluates the degree of airway narrowing by measuring the volume of air that can be exhaled in one breath - CORRECT ANSWER How should a nurse instruct a patient on how to use a peak flow meter? What does it measure? - Clamp catheter to prevent more air from embolizing into circulation - Place pt in trendelenburg position on the left side, causing air to rise and become trapped in right atrium - Administer o2 prn - Notify HCP - CORRECT ANSWER For a patient suspected of having an air embolism, what are the proper nursing interventions to prevent further patient decline? This position will cause any existing air to rise and become trapped in the right atrium - CORRECT ANSWER For a patient suspected of having an air embolism, why is it best for them to be placed in trendelenburg on the left side? - Cleanse mouth with NS after meals and hs - Use soft bristled tooth brush - Apply prescribed viscous lidocaine to alleviate oral pain - Use water soluble lubricating agents to moisten mouth tissues - Avoid hot liquids/spicy/acidic foods - CORRECT ANSWER For patients diagnosed with oral mucositis, inflammation or ulceration of the oral mucosa, what is proper nursing care? 8-12 times/day - CORRECT ANSWER How often will most newborns want to breast feed? 15-20 minutes/breast - CORRECT ANSWER Approximately how often should an infant be fed on each breast? The periumbilical or epigastric area, slightly left of midline - CORRECT ANSWER Where can an abdominal aneurysm be auscultated? Rheumatic fever - CORRECT ANSWER If strep throat is left untreated, what extremely dangerous complication can develop? High fowler's position - CORRECT ANSWER What is the proper positioning for a paracentesis? The patient's arm should be raised above head ob affected side; if possible, HOB should be raised 30-60 degrees to reduce risk of injury to diaphragm - CORRECT ANSWER Upon insertion of a chest tube, how should the patient be positioned? On the right side for a minimum of 2 hours & then supine for an additional 12-14 hours - CORRECT ANSWER How should a patient be positioned after a liver biopsy and for how long? - Influenza injection - Tdap - CORRECT ANSWER What immunizations are appropriate to get during pregnancy? - Influenza nasal spray - MMR - Varicella - CORRECT ANSWER What immunizations are NOT appropriate to get during pregnancy? Phenazopyridine hydrochloride (Pyridium) - CORRECT ANSWER Defined as a urinary analgesic prescribed to relieve pain and burning associated with a UTI - Body secretions will turn bright red-orange (Urine, sweat, tears) & it is normal - Avoid contact lenses while taking medication (They will stain) - CORRECT ANSWER For patients prescribed Pyridium, what are important nursing education points to discuss with the patient? - Fever - Chills - Tachycardia - Uterine tenderness - Foul smelling lochia - CORRECT ANSWER What are s/s of postpartum endometritis? - Haemophilus influenzae type b (Hib) - Hepatitis B - Pneumococcal conjugate (PCV) - Influenza IM - CORRECT ANSWER What are examples of vaccines that are not live? - Dyspnea - Hypoxemia - Tachypnea - Cough - Chest pain - Hemoptysis - Tachycardia - Syncope - Hemodynamic instability - CORRECT ANSWER What are s/s of an impending PE? To main strict NPO status to avoid additional gallbladder stimulation - CORRECT ANSWER For a patient diagnosed with cholelithiasis who is actively vomiting, what is the priority nursing action and why? Progesterone - CORRECT ANSWER During pregnancy, what hormone causes the esophageal sphincter to relax, leading to pyrosis (Heartburn)? S3 - CORRECT ANSWER This extra heart sound is also known as a ventricular gallop Heart failure or FVE - CORRECT ANSWER What does auscultation of S3 upon assessment most likely indicate? Pain management and pulmonary hygiene (Coughing, deep breathing) - CORRECT ANSWER For a patient that experienced multiple rib fractures, what nursing interventions are vital for this patient? "CN's 3, 4 & 6 make the eyes do tricks" - III - IV - VI - CORRECT ANSWER A nurse moves a finger in a horizontal and vertical motion in front of the client's face while directing the client to follow the finger with the eyes. What CN's are being tested? (Think saying learned in ATI) Psychomotor retardation - CORRECT ANSWER This clinical symptom of major depressive disorder include slowed speech, decreased movement and impaired cognitive function - Meats (Beef, lamb, liver, chicken, pork) - Shellfish (Oysters, clams, shrimp) - Eggs - Green leafy veggies - Dried fruits - Beans - Brown rice - Oatmeal - CORRECT ANSWER What are examples of foods that are high in iron? Pinworm infection - CORRECT ANSWER A child is experiencing anal itching and trouble sleeping. What infection does the nurse suspect? The patient should be positioned with the good lung down which promotes better ventilation and drainage of bad lung - CORRECT ANSWER For a patient with lobular pneumonia, how should they be positioned in bed to prevent drops in pulse oximetry and increased respiratory distress? Epiglottitis - CORRECT ANSWER Pediatric ailment that ewers to inflammation of the epiglottis that may result in a life threatening airway obstruction Haemophilus influenzae type b (Hib) - CORRECT ANSWER What is the most common causative agent for epiglottitis? - Drooling - Dysphonia (Trouble speaking) - Dysphagia (Trouble swallowing) - Distressed airway (Stridor) - CORRECT ANSWER What are the 4 D's r/t epiglottitis? Epiglottitis - CORRECT ANSWER This pediatric ailment presents as tripoding (Sitting up and leaning forward) with inspiratory stridor Blurred vision - Optic neuritis is possible to develop - CORRECT ANSWER For a patient diagnosed with TB being treated with ethambutol (Myambutol), what AE does the nurse need to instruct the patient to report to HCP? Parietal lobe - CORRECT ANSWER This area of the brain integrates somatic and sensory input & injury to this area can result in a deficit with sensation Frontal lobe - CORRECT ANSWER This area of the brain controls higher-order processing, such as executive function and personality. Injury to this area results in behavioral changes Temporal lobe - CORRECT ANSWER This area of the brain integrates visual and auditory input and past experiences. Injury to this area may result in inability for the client to understand verbal/written language Occipital lobe - CORRECT ANSWER This area of the brain registers visual images. Injury to this area can result in a deficit with vision CN VII - Facial nerve - CORRECT ANSWER Bell's palsy is characterized by facial paralysis resulting from inflammation of which CN? - Inability to completely close eye on affected side - Flattening of the nasolabial fold on the side of paralysis - Inability to smile/frown symmetrically - Alteration in tear production due to eyelid muscle weakness - Loss of taste on the anterior 2/3 of tongue - CORRECT ANSWER What are clinical manifestations of Bell's palsy? - Oxygen - Fluids - Maintenance of body temperature (Homeostasis) - Sleep - Proper elimination - Adequate shelter - CORRECT ANSWER According to Maslow, what are considered physiological needs that are vital for survival? - Take BP sitting then standing - CORRECT ANSWER What is the best way the nurse can assess for autonomic neuropathy? Involuntary body functions: - BP - HR - Perspiration - Sexual function - Digestion - CORRECT ANSWER What is the autonomic nervous system in control of? - PaCO2 > 45 mm Hg - PaO2 < 60 mm Hg - Respiratory acidosis - Paradoxical breathing (Abnormal inward movement of chest on inspiration and outward movement on expiration) - Mental status changes (Restlessness, confusion) - Absence of wheezing and silent chest - Single word dyspnea - CORRECT ANSWER What are clinical manifestations of impending respiratory failure? 1.5-2.5 mEq/L - CORRECT ANSWER What is the therapeutic level of magnesium? 4-7 mEq/L - CORRECT ANSWER What is the therapeutic level of magnesium sulfate for a pre-eclamptic patient? 4,000-11,000/mm3 - CORRECT ANSWER What is the normal range for WBCs? Neutropenia - CORRECT ANSWER Considered an absolute neutrophil count below 1,000/ mm3 Severe neutropenia - CORRECT ANSWER Considered an absolute neutrophil count below 500/mm3 Take a warm shower/bath upon awakening; Heat decreases stiffness and promotes muscular relaxation/mobility - CORRECT ANSWER What is an intervention to prevent morning stiffness of affected joints in patients with RA? Epidural hematoma - CORRECT ANSWER Defined as the accumulation of blood between the skull bone and dura mater Epidural hematoma - CORRECT ANSWER This kind of hematoma results from an arterial bleed, therefore it develops quickly and is an emergency Subdural hematoma - CORRECT ANSWER This kind of hematoma results from a venous bleed, therefore it develops slowly and is an extreme emergency Offer an elbow for guidance and walk slightly ahead of the patient - CORRECT ANSWER When walking with a client who is blind, what is proper protocol? Ensure cuff is inflated - CORRECT ANSWER For clients with a tracheostomy on mechanical ventilation, what nursing intervention is vital to prevent aspiration? Hemorrhagic stroke - CORRECT ANSWER This type of stroke occurs when a blood vessel ruptures in the brain and causes bleeding into the brain tissue or subarachnoid space - Too much stimulation (Bright room, frequent visitors) - Straining bowel/bladder - Coughing/sneezing - Exertion - HOB supine - Head off from midline position - CORRECT ANSWER What types of activities/interventions increase ICP? - Vasodilation: Hypotension and edema - Tachycardia - Stridor/hoarseness - Bronchospasm: Wheezing - Urticarial rash - Pruritus - N&V, abdominal pain - CORRECT ANSWER What are the clinical manifestations of an anaphylactic reaction? Pleural effusion - CORRECT ANSWER Defined as an abnormal collection of fluid in the pleural space between the parietal and visceral pleurae that prevents the lung from expanding fully Rationalization - CORRECT ANSWER This defense mechanism uses excuses to explain away threatening circumstances Displacement - CORRECT ANSWER This defense mechanism transfers thoughts and feelings toward one person or object onto another person or object Regression - CORRECT ANSWER This defense mechanism involves returning to a previous level of development Introjection - CORRECT ANSWER This defense mechanism involves taking on the qualities or attitudes of others without thought to examination Reaction formatio - CORRECT ANSWER This defense mechanism involves behaving in a manner or expressing a feeling of opposite of one's true feelings Repression - CORRECT ANSWER This defense mechanism involves keeping unacceptable thoughts or traumatic events buried in the unconscious Sublimation - CORRECT ANSWER This defense mechanism involves transforming unacceptable thoughts or needs into acceptable actions Rationalization - CORRECT ANSWER What defense mechanism is this: "I did poorly on the test because the questions were so tricky." Displacement - CORRECT ANSWER What defense mechanism is this: A person who is angry with a boss comes home and yells at their spouse. Regression - CORRECT ANSWER What defense mechanism is this: An adult has a "temper tantrum" when stuck in traffic Introjection - CORRECT ANSWER What defense mechanism is this: A person may take on the political views of a famous admired actor Reaction formation - CORRECT ANSWER What defense mechanism is this: A parent who is resentful of an "unplanned" child becomes overprotective of that child Repression - CORRECT ANSWER What defense mechanism is this: A person who was raped cannot recall the event Sublimation - CORRECT ANSWER What defense mechanism is this: A person may turn to voting to deal with aggression Projection - CORRECT ANSWER This defense mechanism involves feeling uncomfortable with an impulse or feeling and easing the anxiety by assigning it to another person Projection - CORRECT ANSWER What defense mechanism is this: A husband with thoughts of infidelity who then accuses his wife of being unfaithful NG tubes - CORRECT ANSWER What nursing intervention is contraindicated post bariatric surgery due to potential disruption of the surgical site? Diverticulitis - CORRECT ANSWER When diverticula become inflamed, the patient is diagnosed with what? Diverticular disease - CORRECT ANSWER Defined as a disease of the colon in which sac-like protrusions in the large intestine are caused by chronic increased intra-abdominal pressure and/or chronic constipation Vital infections such as varicella or influenza - CORRECT ANSWER For children who develop Reye syndrome, what infections did they most likely have? 18-20 weeks in primigravidae 14-16 weeks in multigravidae - CORRECT ANSWER In pregnant patients, when is quickening (awareness of fetal movements) detected? Be DR HIP - B: Hepatitis B - D: DTaP - R: RV - H: Hib -I: IPV - P: PCV - CORRECT ANSWER What are the routine immunizations for a 6 month client? (Think acronym) Be DR HIP - B: Hepatitis B - D: DTaP - R: RV - H: Hib -I: IPV - P: PCV - CORRECT ANSWER Related to immunizations for a 6 month old patient, what does the acronym Be DR HIP stand for? - Suicide - Homicide - Accident - Within 24 hours of admission of facility - CORRECT ANSWER Under what circumstances is consent from family for an autopsy not required? We - waist circumference Better - blood pressure Think - Triglycerides High - HDL Glucose - Glucose - CORRECT ANSWER Related to metabolic syndrome, what does the nematic "We Better Think High Glucose" stand for? Raynaud's phenomenon - CORRECT ANSWER During a typical episode of this disease, digital arteries constrict and blood flow is impaired, causing the skin to turn pale and then blue, and feel numb and cold Potassium - CORRECT ANSWER This electrolyte is released when cellular damage occurs with burns - Muscle weakness - ECG changes (Tall, peaked t-waves, shortened QT interval) - Cardiac arrhythmias - CORRECT ANSWER What are the clinical manifestations of hyperkalemia? Myopia - CORRECT ANSWER Defined as nearsightedness - Headaches - Dizziness - Difficulty seeing objects clearly - Poor performance in school - Rubbing of eyes - Frequent blinking or squinting - CORRECT ANSWER What are the clinical manifestations and parent reports of children suspected of having myopia (Nearsigntedness)? Hirschsprung disease (HD) - CORRECT ANSWER This congenital disease occurs when a child is born with some sections of the distal large intestine missing nerve cells, rendering anal sphincter unable to relax; No peristalsis occurs & no stool is passed Ototoxicity - CORRECT ANSWER What is a major AE of high dose IV furosemide (Lasix) administration? 4 mg/min (Over 40 min) - CORRECT ANSWER For furosemide doses > 120 mg, what should the dosage/min not exceed? Opioids & benzos - CORRECT ANSWER Neonate abstinence syndrome is typically caused by abuse of what drug classes during pregnancy? - ANS: Stuffy nose, sweating, frequent yawning and sneezing - CNS: Irritability, restlessness, high pitched crying, abnormal sleep pattern, hyperactive reflexes - GI: Poor feeding, vomiting & diarrhea - CORRECT ANSWER What are clinical manifestations of neonate abstinence syndrome? - Supine positioning - Instruct pt to bear down & exhale - Apply air occlusive dressing - Pull line out cautiously (Do NOT pull harder if there is resistance) - CORRECT ANSWER What are proper nursing interventions when discontinuing a central venous catheter to prevent an air embolism from forming? IV 50% dextrose and regular insulin - CORRECT ANSWER For a patient with advanced kidney disease with a serum potassium of 7.1 and a serum creatinine of 4.5 with NO CURRENT ECG changes, what is the priority for the nurse to administer? Calcium gluconate IV followed by insulin/dextrose IV - CORRECT ANSWER For a patient with advanced kidney disease with a serum potassium of 7.1 and a serum creatinine of 4.5 with ECG CHANGES, what is the priority for the nurse to administer? - EPS - Tardive dyskinesia (TD) - Protruding and twisting of tongue - Lip smacking - Puffing of cheeks - Frowning/blinking of eyes - Twisting fingers - Twisted/rotated neck (torticollis) - CORRECT ANSWER What are the adverse effects of Metoclopramide (Reglan)? (Think antipsychotics) CCB - Non-dihydropiridine (Verapamil, Diltiazem) - CORRECT ANSWER This class of blood pressure medications has a direct effect on the HR and should be held for HR < 60 bpm Greater than or equal to 20 bpm increase - CORRECT ANSWER Positive orthostatic vital signs is a rise in pulse of how many bpm? Before - CORRECT ANSWER For patients diagnosed with CF, should bronchodilator treatments be administered before or after chest physiotherapy? Exophthalmos - CORRECT ANSWER A complication of hyperthyroidism defined as a protrusion of the eyeballs caused by increased orbital tissue expansion and can be irreversible - Maintaining HOB in raised position to facilitate drainage from periorbital area - Use of artificial tears to moisten eyes - Taping eyelids shut during sleep if they do not close on their own - Educate on importance of regular visits to ophthalmologist - Educate on importance of anti-thyroid medications - Restrict salt intake - Encourage smoking cessation - Use dark glasses to prevent irritation to eyes - Do eye ROM exercises - CORRECT ANSWER What is proper nursing management for patients experiencing the complication of Graves disease, exophthalmos? 30 mL - CORRECT ANSWER How many mL in one ounce? 2/3 inflated - CORRECT ANSWER When administering a non-rebreather mask, the oxygen flow rate must be high enough to keep the reservoir bag how inflated? - Finger tapping (tapping each finger to that hand's thumb) - Rapid alternating movements (Rapid supine and pronation) - Finger to nose testing - Heel to shin testing (Run heel down opposite shin) - CORRECT ANSWER What are proper nursing interventions to assess proper cerebellum functioning? Beta Blockers (BBs) - CORRECT ANSWER What blood pressure medication class can actually worsen heart failure symptoms by decreasing normal compensatory sympathetic nervous system responses and myocardial contractility Systemic lupus erythematosus (SLE) - CORRECT ANSWER Defined as an autoimmune disorder in which an abnormal immune response leads to chronic inflammation in different parts of the body Kayexalate - CORRECT ANSWER What is the trade name for sodium polystyrene sulfonate Sodium polystyrene sulfonate - CORRECT ANSWER What is the generic name for Kayexalate? Request a meeting with the palliative care team and the parents to discuss end of life choices - CORRECT ANSWER For an infant diagnosed with Edward's syndrome, what is an appropriate nursing action at birth? - Wear gloves when handling cold objects - Dress in warm layers in cold weather - Avoid extreme abrupt weather changes (Cold to hot) - Avoid vasoconstriction drugs (Cocaine, amphetamines, ergotamine, pseudoephedrine) - Refrain from tobacco products - Implement stress management strategies (Toga, tai chi) - CORRECT ANSWER For patients diagnosed with Raynaud phenomenon, what are proper nursing interventions/teaching points to educate the client about? CCBs- relax arteriole smooth muscle and prevent recurrent episodes - CORRECT ANSWER For patients diagnosed with Raynaud phenomenon, what pharmacologic remedy may be prescribed if non-pharmacological methods are unsuccessful? 1 gram = 1 mL of weight - CORRECT ANSWER One gram of weight is equal to how much in mL? - Blowing up balloons causes lip swelling - Hands itch after wearing rubber gloves - Allergy to bananas, avocados, tomatoes, chestnuts, kiwis, potatoes, peaches, grapes and apricots - CORRECT ANSWER Which patient reports would indicate a possible latex allergy? Shellfish - CORRECT ANSWER Allergy to what food group indicates a potential reaction to iodinated contrast dye used during CT scans, catheterizations and angiograms? Distal to the current site - CORRECT ANSWER If the nurse inevitably needs to draw blood from a new site of an extremity with a current IV infusion, where should the blood draw be done? Biliary obstruction - CORRECT ANSWER A patient reporting light gray or "Clay colored," stool is most likely suffering from what? Ulcerative colitis or infectious colitis - CORRECT ANSWER A patient reporting mucus or pus visible in their stool is most likely suffering from what? Chronic pancreatitis - CORRECT ANSWER A patient reporting greasy, foamy, foul smelling or fatty stool is most likely suffering from what? Upper GI bleed - CORRECT ANSWER A patient reporting black tarry stools is most likely suffering from what? Lower GI bleeding - CORRECT ANSWER A patient reporting bright red bloody stools is most likely suffering from what? Cervical cancer - CORRECT ANSWER The occurrence of this disease is higher among hispanics, american indians and african americans African Americans - CORRECT ANSWER What ethnic group has the highest incidence of hypertension in the world Toxic epidermal necrolysis - CORRECT ANSWER Defined as an acute skin disorder, most commonly associated with a medication reaction, resulting in widespread erythema, blistering, epidermal shedding, keratoconjunctivitis and skin erosion; Severe form of SJS Check the tightness by allowing 1 finger to fit underneath the ties - CORRECT ANSWER When caring for a patient with a tracheostomy, what is an effective way to ensure adequate tightness of the ties? - Assess skin for flushing and red rash on face and torso (Red man syndrome) - Infuse medication over at least 60 minutes - Monitor BP during infusion - Observe IV site q30min for pain, redness and swelling - CORRECT ANSWER For patients prescribed Vancomycin, what are proper nursing assessments during administration? Infuse medication over at least 60 minutes - CORRECT ANSWER At what rate does Vancomycin need to be administered to prevent Red Man Syndrome? Red man syndrome - Infusion is too fast, slow it down - CORRECT ANSWER A patient is receiving a Vancomycin infusion and begins to develop skin flushing and a red rash on the face and torso. What does the nurse suspect? Why did this occur? Testicular enlargement - CORRECT ANSWER What is the first sign of pubertal changes that occur in boys? Ebola - CORRECT ANSWER Defined as an extremely contagious disease with a high mortality rate; Also known as hemorrhagic fever - Standard - Contact - Droplet - Airborne * Impermeable gown/coveralls, N95 respirator, full face mask, doubled gloves with extended cuffs, single-use boot covers, single-use apron - CORRECT ANSWER What type of isolation precautions is required for a patient diagnosed with Ebola? - Impermeable gown/coveralls - N95 respirator - Full face mask - Doubled gloves with extended cuffs - Single-use boot covers - Single-use apron - CORRECT ANSWER What PPE is required when caring for a patient with Ebola? Visitors are prohibited unless absolutely necessary for the client's well-being - CORRECT ANSWER What is the visitor policy for client's diagnosed with Ebola? Hypothyroidism - CORRECT ANSWER This endocrine disorder is characterized by high TSH levels and low T3 and T4 levels Hyperthyroidism - CORRECT ANSWER This endocrine disorder is characterized by low TSH levels and high T3 and T4 levels - Constipation - Cool/pale skin - Brittle nails/hair - Hair loss - Bradycardia - Joint pains/muscle aches - Dementia/depression due to mental slowing - Cold intolerance - Weight gain - Forgetfulness - CORRECT ANSWER What are the clinical manifestations of hypothyroidism? (Think slow) - Diarrhea - Warm, moist skin - Smooth, silky hair - Tachycardia/palpitations - Heat intolerance - Weight loss - CORRECT ANSWER What are the clinical manifestations of hyperthyroidism? (Think fast) 1) Place client in semi recumbent position 2) Have client breathe normally 3) Determine SBP using manual cuff 4) Inflate BP cuff to at least 20 mm Hg above previously measured SBP 5) Deflate cuff slowly, noting first korotkoff sound during expiration along with the pressure 6) Continue to slowly deflate cuff until you hear sounds throughout inspiration/expiration 7) Determine difference between two measurements, equaling amount of paradox - CORRECT ANSWER What is proper procedure for measurement of pulses paradoxus? Hyponatremia - CORRECT ANSWER This electrolyte imbalance can lead to seizures, confusion or other neurologic complications - Fluid restriction to < 1000 mL/day - Oral salt tablets - Hypertonic saline (3%) - Vasopressin - CORRECT ANSWER What is the treatment for SIADH? VAP - Ventilator associated pneumonia - CORRECT ANSWER Defined as the second most common health-care associated infection in the US, associated with increased mortality, hospitalization and length of stay - Purulent sputum - Positive sputum culture - Leukocytosis - Elevated temperature (> 100.4 F) - New progressive pulmonary infiltrates on chest x-ray - CORRECT ANSWER What are the clinical manifestations of VAP (Ventilator associated pneumonia)? Psoriasis - CORRECT ANSWER Defined as a chronic autoimmune disease that causes a rapid turnover of epidermal cells - Avoid triggers (Stress, trauma, infection) - Topical therapy (Corticosteroids, moisturizers) - Phototherapy (Ultraviolet light) - Systemic medications (Methotrexate, infliximab) - Avoidance of alcohol - CORRECT ANSWER Though there is no cure for psoriasis, what is proper nursing management? Cytotoxic and biologic agents such as: - Methotrexate - Infliximab - CORRECT ANSWER What pharmacologic agents are shown to assist in treatment of psoriasis? Scleroderma - CORRECT ANSWER Defined as an overproduction of collagen that causes tightening and hardening of the skin and connective tissue; Progressive and no current cure Renal crisis - CORRECT ANSWER Defined as a life threatening complication of scleroderma that causes malignant hypertension; Potentially fatal Sims' position - to facilitate movement of co2 utilized during surgery to fill the abdominal cavity - CORRECT ANSWER For patients post laparoscopic cholecystectomy, what is proper positioning post op and why? Ileostomy - CORRECT ANSWER Defined as a surgically created opening (stoma) in the abdomen that connects the small intestine to the external abdomen (Think placement) Ileostomy - CORRECT ANSWER In this type of ostomy, stool from the small intestine bypasses the colon and exits through external opening of abdomen via ostomy bag Liquid stool - when stool bypasses the colon, fluid and electrolyte absorption and vitamin k production do not occur, resulting in liquid stool - CORRECT ANSWER For patients with an ileostomy, what kind of stool will form and why? - High fiber (popcorn, coconut, multigrain bread) - Stringy veggies (Celery, broccoli, asparagus) - Seeds or pits (Strawberries, rasberries, olives) - Edible peels (Apple slices, cucumber, dried fruit) - CORRECT ANSWER What diet items should be absolutely avoided for a patient with an ileostomy? Calcium - CORRECT ANSWER The parathyroid gland regulates what serum electrolyte levels? Tamiflu - CORRECT ANSWER What is the trade name for Oseltamivir? Oseltamivir - CORRECT ANSWER What is the generic name for Tamiflu? Claratin - CORRECT ANSWER What is the trade name for Loratadine? Loratadine - CORRECT ANSWER What is the generic name for Claratin? Promethazine - CORRECT ANSWER What is the generic name for Phenergan? Phenergan - CORRECT ANSWER What is the trade name for Promethazine? Cerebrospinal fluid (CSF) - CORRECT ANSWER Leakage of this fluid can confirm skull fracture has occurred and transversed the dura - Raccoon eyes & Battle's sign; Indicates leakage of CSF - CORRECT ANSWER What are two clinical indications that confirm a skull fracture? What does this indicate? - Remove clothing & jewelry to prevent constriction - DO NOT MASSAGE, rub or squeeze area involved (Will lead to further tissue damage) - Immerse affected area in heated water (98.6-102.2F) - Provide analgesia during rewarming (Painful) - Monitor for compartment syndrome - CORRECT ANSWER What does the treatment of frostbite consist of? Thoracentesis - CORRECT ANSWER This procedure is commonly used to treat pleural effusions - Pneumothorax - Bleeding - Tension pneumothorax - Infection - CORRECT ANSWER What are some complications of thoracentesis? - Increased RR - Increased respiratory effort - Respiratory distress - Low pulse ox - Absent breath sounds on affected side - CORRECT ANSWER What are the clinical manifestations of a patient developing a pneumothorax? - Take medication at the same time daily - Report for periodic blood tests to assess for therapeutic effect - Avoid contact sports, vigorous teeth brushing and use of razor blades - Avoid aspirin and other NSAIDs - Avoid changing eating habits (Increase of leafy greens) - Do NOT take vitamin K supplements if on warfarin - Consult HCP before starting/stopping any herbal supplements (Ginko, garlic etc) - Wear a medical alert bracelet indicating what anticoagulant is being taken - CORRECT ANSWER What are some important patient teaching points related to anticoagulation therapy? - Re- experiencing the traumatic event - Flashbacks - Nightmares - Feelings of distress in reaction to reminders - Avoiding reminders of the trauma - Increased anxiety - Emotional arousal - CORRECT ANSWER What are some clinical manifestations of PTSD? - Pacemaker malfunction; Immediate skilled intervention required - CORRECT ANSWER What does a client with a demand pacemaker set at 70/min who has a ventricular rate of 65/min indicate? Osteomyelitis - CORRECT ANSWER Defined as a serious bone infection that requires long-term treatment with antibiotics Rapid-acting insulin - onset of 10-15 minutes - CORRECT ANSWER Insulin Aspart (NovoLOG) is what kind of insulin? What is the onset? Hematoma formation - Maintain patent airway - CORRECT ANSWER Swelling of the neck and increased pain post thyroidectomy may indicate what post op? What is the priority nursing action? Mucinex - CORRECT ANSWER What is the trade name for Guaifenesin? Guaifenesin - CORRECT ANSWER What is the generic name for Mucinex? Claritin - CORRECT ANSWER What is the trade name for Loratadine? Loratadine - CORRECT ANSWER What is the generic name for Claritin? NSAIDs & Non-cardioselective BB's - CORRECT ANSWER What two medication classes can be a potential problem for patients with asthma? Angioedema - CORRECT ANSWER Defined as rapid swelling of the lips, tongue, throat, dace and larynx during ACE/ARB inhibitor therapy Blood glucose control over a period of 2-3 months - CORRECT ANSWER For patients with diabetes, the Hemoglobin A1C test measures what? Ventricular trigeminy - CORRECT ANSWER During this irregular heart rhythm, premature ventricular contractions (PVCs) occur every third heartbeat - Hypoxia - Electrolyte imbalances - Emotional stress - Fever - Stimulants (Caffeine, drugs) - Exercise - CORRECT ANSWER What are the most common causes of PVC's? - Adequate nutrition - Adequate circulation - Age - CORRECT ANSWER What 3 factors does bone healing depend on? Cover the insertion site with something clean & occlusive, such as a GLOVED HAND, to prevent air from entering the pleural cavity - CORRECT ANSWER When sudden dislodgment of a chest tube occurs & a sterile occlusive dressing is unavailable, what is the next best nursing intervention? S/S of hyperglycemia - CORRECT ANSWER When caring for a patient receiving TPN, what is it extremely important for the nurse to monitor for? - Polyuria - Polydipsia - Polyphagia - Headaches - Blurred vision - CORRECT ANSWER What are s/s of hyperglycemia? - EPS: Parkinsonism, akathisia, shuffling gait - Neuroleptic malignant syndrome (NMS) - Anticholinergic effects: BUDCAT - Orthostatic hypotension - Sedating effects: Drowsiness, hypersomnia - CORRECT ANSWER What are potential AE of atypical antipsychotics, such as Seroquel & Zyprexa? No longer than 30 minutes - Should return blood to blood bank - CORRECT ANSWER No longer than what amount of time can blood products remain at room temperature after picking them up from the blood bank? What is proper nursing action if in this situation? q5-10days - CORRECT ANSWER How often should an ostomy appliance be changed? Preeclampsia - CORRECT ANSWER This pregnancy condition is defined as new-onset HTN after 20 weeks gestation accompanied with proteinuria or signs of end-organ damage - Net onset HTN - Proteinuria > 300mg/24hr - Signs of end organ damage - Headache - Visual changes - Pitting pedal edema - CORRECT ANSWER What are common clinical manifestations of preeclampsia? The nurse should complete administration of the blood products and allow one hour of observation post administration, then begin initiation of Amphoteracin B; This is done because adverse effects of both blood products and the anti fungal medication are too similar to differentiate between - CORRECT ANSWER The nurse has orders to administer both blood products and Amphoteracin B. How should the nurse go about administering both of these medications? Why? - Remain with client - Protect airway using head tilt chin lift OR jaw thrust if spinal injury suspected - Deliver rescue breaths using bag-valve-mask with 100% o2 until re-intubation is achieved - CORRECT ANSWER If accidental extubation occurs when repositioning/turning a sedated patient in the ICU, what are vital nursing actions? Headache due to vasodilation of cranial vessels - CORRECT ANSWER After administering IV nitroglycerin to a patient with angina, what is an expected side effect from medication therapy and why? Tubing became disconnected OR there is an air leak - CORRECT ANSWER What could a low pressure alarm on a ventilator indicate? - Tubing is kinked - Obstruction (secretions) in tube - Pt is biting endotracheal tube (Fighting ventilator) - CORRECT ANSWER What could a high pressure alarm in an endotracheal or tracheostomy tube indicate? - Falls - Mislabeled laboratory specimens - Medication administration errors - Failure to communicate pertinent patient information to HCP - CORRECT ANSWER What are examples of client incidents that would require completion of an incident report? Buccal mucosa & conjunctivae - CORRECT ANSWER Where is the best area to assess for petechiae in dark skinned clients? Palms of hands and soles of feet - CORRECT ANSWER Where is the best area to assess for jaundice in dark skinned clients? - Activity intolerance - Barrel chest - Hyperresonance on percussion - Use of accessory muscles - Pursed lip breathing - Placement in tripod position - CORRECT ANSWER What are assessment findings found in patients with emphysema? T- Toxoplasmosis O - Other: Parovirus, varicella R - Rubella C - Cytomegalovirus H - Herpes simplex virus - CORRECT ANSWER What does TORCH stand for related to gestational infections that can cause fetal abnormalities - Move patient out of large bodies of water - Dry chest area before applying pads - Remove transdermal medication patches before application of AED - CORRECT ANSWER Before applying AED pads to a patient in the community setting, what are nursing considerations to have? - Hyperphosphatemia - Hyperkalemia - Hyperuricemia - Hypocalcemia - CORRECT ANSWER What are the severe electrolyte abnormalities seen in chemotherapy induced tumor lysis syndrome? Tumor lysis syndrome - CORRECT ANSWER Defined as an oncologic emergency occurring when cancer treatment successfully kills cancer cells, resulting in release of intracellular components, such as electrolytes - Fever or hypothermia - Poor feeding - Vomiting - Altered LOC (Irritable, restless) - Increased ICP (Bulging fontanelle) - CORRECT ANSWER What are s/s of meningitis in an infant? Laryngotracheobronchitis aka Croup - CORRECT ANSWER In pediatric clients, an occasional barking cough without stridor is most likely what? - Green, black, white or dusky coloration - Accumulations of air - Evidence of clotting or presence of inclusions - Malodor - CORRECT ANSWER Upon examination of an IV bag containing blood products, what are examples of indications of contaminated blood? Contamination - Return blood to blood bank & obtain a new bag - CORRECT ANSWER Upon examination of an IV bag containing blood products, what does a large air bubble at the top of the bag indicate? What is the proper nursing action? - Female gender - Older adults - Patiens with DM experiencing neuropathy - CORRECT ANSWER Patients with what two characteristics/history will have atypical presentation of MI symptoms? - Described as chest pressure, heaviness, tightness - Pain may radiate to jaw, arm, back or upper abdomen - CP lasting more than 30 minutes - CP not improved with rest or position changes - CP worsens with exertion - CORRECT ANSWER What are the TYPICAL clinical manifestations of impending MI? DMARD: Immunosuppressant Educate about: - Increased risk for infection - Bone marrow suppression - Avoid large crowds - Avoid known sick contacts - CORRECT ANSWER What drug class is Cyclosporine in? What does the nurse need to educate patients prescribed this about? Constipation - CORRECT ANSWER What is an AE of the gastrointestinal tract upon taking calcium channel blockers, specifically Verapamil? - Bradycardia - Irregular respirations - HTN with a widening pulse pressure - CORRECT ANSWER In relation to increased ICP, what defines the Cushing triad? Singed facial hair - CORRECT ANSWER Upon assessment of a patient, what assessment finding can indicate possible smoke inhalation injury? - Hypotension - Bradycardia - Dizziness - CORRECT ANSWER What are s/s of pacemaker malfunction? - Antiarrhythmics (Sotolol, amiodarone, ibutilide, dofetilide) - Macrolide antibiotics (Erhtyromycin, azithromycin) - Electrolyte abnormalities (Hypokalemia, hypomagnesemia) - CORRECT ANSWER What are examples of medications/medical conditions that can prolong QT interval and precipitate an episode of torasdes de points? Supine - CORRECT ANSWER For patients in restraints, what position is contraindicated due to increased risk for aspiration? - Side lying - Semi Fowler - High Fowler - CORRECT ANSWER What 3 positioning techniques are recommended for patients in restraints? - Hemoglobin > 11 g/dL - Uncontrolled HTN - Increased risk of thrombotic events - CORRECT ANSWER For patients with chronic kidney disease receiving treatment with erythropoietin (Epotein alfa, Epogen), when would therapy be discontinued and why? - Assess BP - Assess Hgb level - CORRECT ANSWER For patients with chronic kidney disease scheduled for erythropoietin therapy, what are proper nursing assessment prior to administration? Uncontrolled HTN - CORRECT ANSWER For patients with chronic kidney disease prescribed erythropoietin, what is a contraindication for therapy with this medication? Hepatic encephalopathy (HE) - CORRECT ANSWER This disease process results from the liver's failure to detoxify the body of ammonia - Cognitive deficits (Confusion, disorientation) - Mental status changes (Sleep disturbances, altered LOC, hypersomnia, coma) - Motor alterations (Asterixis: Flapping hand tremor when arms are extended) - Fector hepaticus (Sweet, musty odor to breath) - CORRECT ANSWER What are the clinical manifestations of patients developing hepatic encephalopathy? [Show More]

Last updated: 7 months ago

Preview 1 out of 90 pages

Add to cart

Instant download

We Accept:

We Accept
document-preview

Buy this document to get the full access instantly

Instant Download Access after purchase

Add to cart

Instant download

We Accept:

We Accept

Also available in bundle (1)

ALL NCLEX Exam (28 SETS) Questions with Verified Answers,100% CORRECT

NCLEX-RN EXAM REVIEW 133 Questions with Verified Answers,NCLEX Exam 116 Questions with Verified Answers, NCLEX Exam 75 Questions with Verified Answers,NCLEX Practice Exam 1|40 Questions with Verified...

By securegrades 7 months ago

$29

28  

Reviews( 0 )

$11.00

Add to cart

We Accept:

We Accept

Instant download

Can't find what you want? Try our AI powered Search

OR

REQUEST DOCUMENT
127
0

Document information


Connected school, study & course


About the document


Uploaded On

Nov 14, 2023

Number of pages

90

Written in

Seller


seller-icon
securegrades

Member since 4 years

117 Documents Sold


Additional information

This document has been written for:

Uploaded

Nov 14, 2023

Downloads

 0

Views

 127

Recommended For You

Get more on EXAM »

$11.00
What is Browsegrades

In Browsegrades, a student can earn by offering help to other student. Students can help other students with materials by upploading their notes and earn money.

We are here to help

We're available through e-mail, Twitter, Facebook, and live chat.
 FAQ
 Questions? Leave a message!

Follow us on
 Twitter

Copyright © Browsegrades · High quality services·